Dispute Resolution MCQS Flashcards

1
Q

The overriding objective includes dealing with cases at proportionate cost, which means…

A) …in ways proportionate to the court’s resources.

B) …in ways proportionate to the amount of money in dispute.

C) …in ways proportionate to the amount of money in dispute, the complexity of the case and other factors

D) …in ways proportionate to the importance of the case to the parties

A

C) …in ways proportionate to the amount of money in dispute, the complexity of the case and other factors

(Well done. This is what CPR 1.1 says. All the factors mentioned in the other answers are potentially relevant when considering proportionality in the overriding objective, but to focus on any one of those points would be to miss the multi-factorial approach which the court must take)

How well did you know this?
1
Not at all
2
3
4
5
Perfectly
2
Q

CPR rules must be followed. The provisions in practice directions…

A) …explain the CPR rules.

B) …must also be followed.

C) …are guidance which show best practice but need not necessarily be followed

A

B) …must also be followed

(Correct. Whilst it is true that the provisions in a practice direction might help you to interpret the CPR, the rules in the PDs must be followed in their own right)

How well did you know this?
1
Not at all
2
3
4
5
Perfectly
3
Q

The court’s approach to case management is one best characterised as….

A) …active on the court’s part, meaning the court will ensure that cases progress appropriately even if the parties do not seek such involvement from the court.

B) …active on the court’s part, meaning the court will act to seek out evidence in order to make a fair determination of the issues in dispute.

C) …responsive to requests from the parties for court intervention and guidance, helping them to move the case along appropriately.

A

A) …active on the court’s part, meaning the court will ensure that cases progress appropriately even if the parties do not seek such involvement from the court

(Well done. This is what active case management means. See also CPR 1.4. The court is responsive to requests from the parties, but it is not only responsive – it takes action of its own accord, and this is really what ‘active’ case management means. Judges in the courts of England and Wales do not generally seek out evidence, rather than determine cases on the basis of the evidence put before them – so this answer would be wrong, and in any event, this is not really what case management is about)

How well did you know this?
1
Not at all
2
3
4
5
Perfectly
4
Q

A claimant is told that the civil standard of proof is on a balance of probabilities. How would you expand this description so that the claimant understands what ‘on a balance of probabilities’ means?

A) That the claimant’s case is more probably true than the defendant’s case

B) That the court can be ‘sure’ that the claimant’s case is correct.

C) That the defendant’s case is unlikely to be true.

D) That the claimant’s case is more probably true than not true

A

D) That the claimant’s case is more probably true than not true

(Well done. This answer is correct. The civil standard of proof is on a balance of probabilities which means that a fact is established if it is more likely than not to have happened, hence the claimant’s case is more probably true than not true)

How well did you know this?
1
Not at all
2
3
4
5
Perfectly
5
Q

In a breach of contract case, which party usually has the burden of proving the duty?

A) Claimant

B) Claimant and defendant

C) Defendant

A

A) Claimant

(Well done. The burden of proving any issue of fact or law generally falls on the party who asserts it. Therefore, as the claimant asserts the duty in order to establish a claim, it will usually fall on the claimant to prove the duty)

How well did you know this?
1
Not at all
2
3
4
5
Perfectly
6
Q

The approach to completing a case analysis should include consideration of….

A) Causation

B) Breach

C) Loss

D) Duty breach causation and loss

E) Duty

A

D) Duty breach causation and loss

(Well done. All four legal elements of a cause of action should be considered as part of a case analysis)

How well did you know this?
1
Not at all
2
3
4
5
Perfectly
7
Q

From when does the limitation period generally start running in contract and tort claims?

A) In both instances, from the date of actionable damage.

B) In both instances, from the date of breach.

C) In the case of contract, from the date of breach. In the case of tort, from the date of actionable damage

A

C) In the case of contract, from the date of breach. In the case of tort, from the date of actionable damage.

(Well done. The limitation period for both contract and tort starts to run from when the cause of action accrues. For contract claims, this will be from the date of the breach of contract whereas for tortious claims, time starts to run from the date of actionable damage)

How well did you know this?
1
Not at all
2
3
4
5
Perfectly
8
Q

How long is the limitation period for contract and tort claims (where none of the more specific provisions apply)?

A) 1 year

B) 3 years

C) 6 years

D) 12 years

A

C) 6 years

(Well done. This is the general rule, but there are specific provisions for personal injury claims and some latent damage claims which must be considered)

How well did you know this?
1
Not at all
2
3
4
5
Perfectly
9
Q

Marks Ltd, a landscaper, agrees to purchase 10 bulk bags of wood chip from a supplier at £150 each. The contract was signed on 8 January 2020 and it was a term of the contract that the wood chip would be delivered to Marks Ltd’s yard in Dorchester on 2 March 2020. It was a further term of the contract that time was of the essence. You ascertain that the wood chip was delivered to your client’s London premises 4 weeks after the contractual date of delivery. The knock-on effect of this was that Marks Ltd has lost a contract with a customer. During the case analysis, you must work out when the limitation period expires. On which of the following dates does the limitation period expire?

A) 22 December 2025

B) 2 March 2026

C) 8 January 2026

A

B) 2 March 2026

(Correct. Under s.5 Limitation Act 1980, the limitation period for a breach of contract claim is effectively 6 years from the date of contractual breach, which is the contractual date of delivery in this case)

How well did you know this?
1
Not at all
2
3
4
5
Perfectly
10
Q

On 1 March 2020, Kelly is injured when a motorist collides with her car. She is immediately aware of her injuries. She exchanges personal details with the motorist that caused the accident. On 1 April 2020, a solicitor advises her that she may have a claim in negligence against the motorist. When will limitation expire?

A) 1 March 2023

B) 1 April 2023

C) 1 March 2026

D) We cannot say on these facts

A

A) 1 March 2023

(Well done. This is a claim for personal injuries. The claim must be brought within 3 years of the latest of when the cause of action accrued, or the date of knowledge. In this case, those 2 dates are the same – the date of knowledge was the same as the accrual of the cause of action – 1 March 2020. It is not necessary for Kelly to know that she has a claim in negligence in order for her to have the requisite knowledge, so the advice from the solicitor is not relevant)

How well did you know this?
1
Not at all
2
3
4
5
Perfectly
11
Q

A litigation claim is being brought against a sole trader called April Carter. April runs a fruit and vegetable shop called ‘Freshfare’ and the claim is in relation to the shop. How should April best be described in the claim?

A) April Carter

B) Freshfare Limited

C) Freshfare LLP

D) April Carter (trading as Freshfare)

E) Freshfare (a firm)

A

D) April Carter (trading as Freshfare)

(Correct; for sole traders, commonly both the individual and trading name will be used when describing a party to the claim and therefore the ‘best’ answer is April Carter (trading as Freshfare). This is particularly the case as the dispute appears to be in relation to her shop)

How well did you know this?
1
Not at all
2
3
4
5
Perfectly
12
Q

What steps can be taken against a child or protected party in proceedings before the appointment of a litigation friend?

A) Make an interim application pre-action

B) Serve a defence

C) Issue and serve a particulars of claim

D) Issue and serve a claim form

D) Make an interim application during proceedings

A

D) Issue and serve a claim form

(Correct. A person cannot make an application against a child or a protected party before proceedings have started or take any step in the proceedings unless the child or protected party has a litigation friend (CPR 21.3). The exceptions are to serve a claim form or to seek the appointment of a litigation friend by way of a court order. Any other step taken in the proceedings where there is no litigation friend has no effect (CPR 21.3(5)).
Make an interim application during proceedings or pre-action, serve a defence or issue and serve a particulars of claim are all incorrect because of CPR 21.3 which limits what steps can be taken before a litigation friend is appointed)

How well did you know this?
1
Not at all
2
3
4
5
Perfectly
13
Q

The claimant is suing Catherine Benson and Ahmed Khan, an accountancy partnership called B&K Accountancy. Who should the claimant bring their claim against?

A) B&K Accountancy (A Firm)

B) Catherine Benson and Ahmed Khan

C) Catherine Benson and Ahmed Khan trading as B&K Accountancy

D) B&K Accountancy LLP

E) Catherine Benson and Ahmed Khan trading as B&K Accountancy is incorrect because it is not a sole trader

A

A) B&K Accountancy (A Firm)

(B&K Accountancy Limited is incorrect because it is not a company.
Catherine Benson and Ahmed Khan trading as B&K Accountancy is incorrect because it is not a sole trader.
Catherine Benson and Ahmed Khan is incorrect because of the contents of 7A PD 5A whereby claims brought against a partnership must be brought against the name under which the partnership carried on business unless it is inappropriate to do so. There is no reason in the brief facts given not to sue in the partnership name. It is sometimes appropriate to sue the partners individually if it seems likely that it will be necessary to seek enforcement of judgment against the partners’ personal assets, as well as the partnership assets, but there is nothing on the facts to suggest this (and in that instance, it is more likely that both the partnership name and the individuals’ names would be used).
B&K Accountancy LLP is incorrect because it is not an LLP)

How well did you know this?
1
Not at all
2
3
4
5
Perfectly
14
Q

Which of the following IS NOT a consequence indicated in the practice direction on pre-action conduct as a possible result of failing to comply with the PD on pre-action conduct or applicable pre-action protocol?

A) The claim could be stayed.

B) The claim could be struck out.

C) The party failing to comply could be penalised in interest.

D) The party failing to comply could be penalised in costs.

A

B) The claim could be struck out

(Well done. Whilst this is theoretically possible (CPR 3.4(2)) it is not specifically mentioned in the practice direction on pre-action conduct, perhaps because alternatives such as a stay are more proportionate sanctions)

How well did you know this?
1
Not at all
2
3
4
5
Perfectly
15
Q

Which of the following is most likely to justify issuing proceedings without full compliance with the rules on pre-action conduct?

A) The value of the claim being so small that compliance would be disproportionate.

B) The defendant refusing to contemplate settlement of the claim.

C) The imminent expiry of a limitation period.

A

C) The imminent expiry of a limitation period.

(Correct. A party cannot be expected to comply with the rules on pre-action conduct if that would lead to proceedings being issued too late from a limitation perspective. In these circumstances, a party should issue proceedings but then apply to court for a stay of the proceedings while they comply with the pre-action rules (PD on pre-action conduct, para 17). In terms of the other answers, compliance is expected in a claim of small value – you could argue that taking steps to resolve a claim without court proceedings is particularly important if the value of the claim is small. The defendant refusing to contemplate settlement of the claim is not a good reason not to comply with the pre-action rules)

How well did you know this?
1
Not at all
2
3
4
5
Perfectly
16
Q

The Practice Direction – Pre-action Conduct and Protocols requires the claimant (in the usual course of events) to write to the defendant with concise details of the claim and for the defendant to respond __________

A) within a reasonable time.

B) within 14 days.

C) within 28 days.

D) as soon as possible.

A

A) within a reasonable time

(The defendant is required to respond within a reasonable time, which could be 14 days in a straightforward case and no more than 3 months in a very complex one (paragraph 6 of the Practice Direction))

How well did you know this?
1
Not at all
2
3
4
5
Perfectly
17
Q

C wishes to bring a personal injury claim against his employer. The claim is valued at £65,000. What is the best advice to give C about whether to issue a claim in the County Court or the High Court?

A) The claim should be issued in the Chancery Division of the High Court.

B) The claim must be issued in the High Court.

C) The claim can be issued in either the County Court or the High Court.

D) The claim should be issued in a specialist division of the County Court.

E) The claim must be issued in the County Court.

A

C) The claim can be issued in either the County Court or the High Court

(Correct. As this is a personal injury claim exceeding £50,000, both the County Court and High Court have jurisdiction. You will need to consider the factors in 7A PD 2 when making a final decision about whether to issue in the County Court or the High Court)

How well did you know this?
1
Not at all
2
3
4
5
Perfectly
18
Q

Which of the following isNOT specified in the CPR as a justification for issuing proceedings in the High Court (when the CPR does not indicate that either the County Court or the High Court is mandatory)?

A) The importance of the outcome to the public.

B) The complexity of the procedures involved.

C) The claimant believes that the claim ought to be dealt with by a High Court judge.

D) The financial value of the claim.

E) The availability of High Court resources (at the time of issue).

A

E) The availability of High Court resources (at the time of issue).

(Correct. The relevant rules do not indicate that this is a potential justification)

How well did you know this?
1
Not at all
2
3
4
5
Perfectly
19
Q

In order to avoid a limitation defence, what must be done before the limitation date?

A) The claim form must be issued.

B) The claim form must be served.

C) The particulars of claim must be served.

D) The claim form and particulars of claim together must be filed and served.

E) The particulars of claim must be filed at court

A

A) The claim form must be issued

(Correct. It is the issuing of the claim form that begins the claim and stops time running for limitation purposes (Limitation Act 1980, CPR 7.2 and 7A PD 5))

How well did you know this?
1
Not at all
2
3
4
5
Perfectly
20
Q

If particulars of claim are not contained in or served with the claim form they must be served…

A) …within 14 days of service of the claim form or within 4 months of the claim being issued (whichever is later).

B) …within 14 days of service of the claim form.

C) …before service of the claim form.

D) …within 14 days of service of the claim form, and also within 4 months of the claim being issued

A

D) …within 14 days of service of the claim form, and also within 4 months of the claim being issued

(Well done. See CPR 7.4(2))

How well did you know this?
1
Not at all
2
3
4
5
Perfectly
21
Q

The deadline for taking the ‘relevant step’ to serve the claim form is 12.00 midnight on the calendar day [how many?] months after the date of issue of the claim form (assuming service within the jurisdiction)?

A) 2

B) 6

C) 4

D) 3

A

C) 4

(Well done. See CPR 7.5(1))

How well did you know this?
1
Not at all
2
3
4
5
Perfectly
22
Q

The deadline for taking the ‘relevant step’ to serve the claim form depends on…

A) …when the claim form was issued.

B) …the method of service to be adopted.

C) …the date of last correspondence with the defendant.

E) …when the limitation period expires.

A

A) …when the claim form was issued.

How well did you know this?
1
Not at all
2
3
4
5
Perfectly
23
Q

An antique furniture dealer commences proceedings against an individual in relation to £25,000 owed by the individual for furniture purchased. The individual has instructed solicitors and the solicitors have provided an address for service. Which of the following is a permitted method of service of the claim form?

A) Service on the business address of the solicitor.

B) Personal service on the individual.

C) Service by post to the usual or last known residence of the individual by post.

D) Service by any of the methods listed in the other answers.

E) Service by depositing the claim form at the usual or last known residence of the individual.

A

A) Service on the business address of the solicitor

(Correct. As the solicitor has been instructed to accept service of proceedings, this is now the only permissible place to serve the claim form (CPR 6.7))

How well did you know this?
1
Not at all
2
3
4
5
Perfectly
24
Q

If a witness statement is served by document exchange, the deemed date of service will be on….

A) The day after the witness statement was left with, delivered to or collected by the DX service provider, provided that day is a business day.

B) The second day after the witness statement was left with, delivered to or collected by the DX service provider, provided that day is a business day.

C) The same day that the witness statement was left with, delivered to or collected by the DX service provider, provided this happens before 4.30pm and that day is a business day.

D) The second day after the witness statement was left with, delivered to or collected by the DX service provider, regardless of whether that day is a business day or not.

E) The day after the witness statement was left with, delivered to or collected by the DX service provider, regardless of whether that day is a business day or not.

A

B) The second day after the witness statement was left with, delivered to or collected by the DX service provider, provided that day is a business day

(Correct. The document served is a witness statement so the rules relating to documents ‘other than a claim form’ apply. These state that for service by DX the deemed date of service will be the second day after taking the necessary steps as listed in the rule to place the document with the DX service provider, provided that day is a business day)

How well did you know this?
1
Not at all
2
3
4
5
Perfectly
25
Q

A defendant agrees to accept service by email. The claim form is then duly served by email. The email is sent on Friday 19 October. The claimant receives notification that it has been read on Monday 22 October. When is the claim form deemed served?

A) Saturday 20 October.

B) Tuesday 23 October.

C) Sunday 21 October.

D) Friday 19 October.

E) Monday 22 October.

A

B) Tuesday 23 October

(Correct. The document served is a claim form (which has different rules to other documents). Deemed date of service will be the second business day after the relevant step to serve was taken, as listed in CPR 7.5(1). For service by email this step is sending the email. Both the first and second days must be business days (ie not weekend or other holiday days) so the first business day is Monday 22 October and the second business day, the date of deemed service, is Tuesday 23 October)

How well did you know this?
1
Not at all
2
3
4
5
Perfectly
26
Q

A witness statement properly addressed is collected by the DX service on Friday 4 March. When is it deemed served?

A) Saturday 5 March

B) Monday 7 March

C) Sunday 6 March

D) Friday 4 March

E) Tuesday 8 March

A

B) Monday 7 March

(Well done. It would be deemed served second day after posting / giving to DX provider, if a business day, and otherwise it would be deemed served the next business day. The DX provider took it on 4 March. The second day after this is 6 March, a Sunday, and not a business day. So it would be deemed served the next business day, which is Monday 7 March)

How well did you know this?
1
Not at all
2
3
4
5
Perfectly
27
Q

A defence is posted (second class) on Thursday 1 November. When is it deemed served?

A) Monday 5 November.

B) Friday 2 November.

C) This is not an acceptable mode of service – it will not be deemed served at all.

D) Thursday 1 November.

E) Saturday 3 November.

A

C) This is not an acceptable mode of service – it will not be deemed served at all

(Correct. Second class post is not an acceptable method of service, only first class post)

How well did you know this?
1
Not at all
2
3
4
5
Perfectly
28
Q

Which of the following lists are all statements of case?

A) Defence and Expert’s report

B) Claim form and Reply to defence

C) Particulars of claim and Witness statement

D) Acknowledgment of service and Defence

A

B) Claim form and Reply to defence

(Correct. Statements of case are key documents by which the parties set out their position in relation to the factual basis of the claim and the claimant’s entitlement to relief. They therefore include the claim form, particulars of claim (if not included in claim form), defence, Part 20 claim and reply to defence)

How well did you know this?
1
Not at all
2
3
4
5
Perfectly
29
Q

The claimant has issued a claim form with attached particulars of claim and served the documents on the defendant. The defendant has filed and served a defence. The claimant realised it has made a mistake in the particulars of claim: it has not set out one of the duties. What should the claimant do?

A) Apply to the court for an amendment

B) Withdraw its claim

C) Tell the defendant

A

A) Apply to the court for an amendment

(Correct. If a statement of case does not comply with necessary content or includes a mistake, it requires formal amendment. This is likely to require the input of the court, giving permission and making further directions to allow for the defendant to amend its defence, etc. There may be costs consequences for the claimant)

How well did you know this?
1
Not at all
2
3
4
5
Perfectly
30
Q

If a statement of case contains a serious false statement, what are the implications for the client who has signed the statement of truth?

A) The case is struck out

B) Nothing. The statement of case is the responsibility of the solicitor.

C) Proceedings for contempt of court

D) Costs sanctions

A

C) Proceedings for contempt of court

(Correct. If the document turns out to contain a false statement and the person signing the statement of truth does not have an honest belief that the statement was true, then proceedings for contempt of court may be brought against the person signing. This can lead to sanctions within the proceedings, fines and/or imprisonment in serious cases)

How well did you know this?
1
Not at all
2
3
4
5
Perfectly
31
Q

Angela Smith and Brian Jones are business partners in a firm which trades under the name ‘AB Partnership’. If suing this partnership, how would the name of this defendant appear on the claim form?

A) AB Partnership Limited

B) (1) Angela Smith (2) Brian Jones

C) AB Partnership LLP

D) AB Partnership (a Firm)

E) Angela Smith and Brian Jones trading as AB Partnership

A

D) AB Partnership (a Firm)

(Correct. As the partners are being sued as a partnership (ie in the name of the firm and not as individuals) and there is nothing to suggest this is anything other than a traditional partnership (ie it is not an LLP), this is the correct format for the name of this defendant)

How well did you know this?
1
Not at all
2
3
4
5
Perfectly
32
Q

It is often appropriate to include the particulars of claim on the claim form for a claim that is a….

A) Against multiple parties

B) Complex contractual dispute

C) Professional negligence claim

D) Personal injury claim for a brain injury

E) Simple debt claim

A

E) Simple debt claim

(Correct. Particulars of claim are usually contained in a separate document, but they can be included in the claim form where they are relatively concise which they are likely to be in a simple debt claim)

How well did you know this?
1
Not at all
2
3
4
5
Perfectly
33
Q

In working out the statement of value in the claim form, the following matters should not be taken into account…

A) Damages, interest, costs, counterclaims, set off, contributory negligence and state benefits the defendant may be liable to pay under the Social Security (Recovery of Benefits) Act 1997 only.

B) Counterclaims, set off, contributory negligence and state benefits the defendant may be liable to pay under the Social Security (Recovery of Benefits) Act 1997 only.

C) Contributory negligence and state benefits the defendant may be liable to pay under the Social Security (Recovery of Benefits) Act 1997 only.

D) Interest, costs, counterclaims, set off, contributory negligence and state benefits the defendant may be liable to pay under the Social Security (Recovery of Benefits) Act 1997 only.

E) Interest and costs only.

A

D) Interest, costs, counterclaims, set off, contributory negligence and state benefits the defendant may be liable to pay under the Social Security (Recovery of Benefits) Act 1997 only.

(Correct. These items should be disregarded for the purposes of the statement of value in the claim form (CPR 16.3(6)))

How well did you know this?
1
Not at all
2
3
4
5
Perfectly
34
Q

In a negligence claim in the High Court, where the claimant claims damages for loss of reputation, which of the following most accurately describes the likely position in relation to interest being claimed in the particulars of claim?

A) A claim for interest on these damages will not be included in the particulars of claim as the amount of these damages will need to be estimated by the court and are not yet outstanding.

B) A claim for interest on these damages can be included in the particulars of claim. This will be pleaded generally and will be based on s69 County Courts Act 1984.

C) A claim for interest on these damages can be included in the particulars of claim. This must include an exact calculation including the percentage rate, period claimed and total to date and will be based on s35A Senior Courts Act 1981.

D) A claim for interest on these damages can be included in the particulars of claim. This will be pleaded generally and will be based on contractual principles.

E) A claim for interest on these damages can be included in the particulars of claim. This will be pleaded generally and will be based on s35A Senior Courts Act 1981

A

E) A claim for interest on these damages can be included in the particulars of claim. This will be pleaded generally and will be based on s35A Senior Courts Act 1981.

(Correct. This is a tort claim (negligence) in the High Court so there is no contractual interest provision that might apply and the Senior Courts Act is appropriate for the statutory claim. This is an unspecified claim, as the damages concerned will need to be estimated by the court, so the interest can be pleaded generally)

How well did you know this?
1
Not at all
2
3
4
5
Perfectly
35
Q

The claimant agreed to provide the defendant with building materials. The contract setting out all terms of the agreement was signed by both the claimant and the defendant. There is now a dispute and the claimant has issued court proceedings against the defendant. What should the claimant attach to the particulars of claim?

A) An expert’s opinion

B) Particulars of an oral agreement

C) A witness statement

D) Particulars of the conduct relied on

E) Written agreement

A

E) Written agreement

(Correct. There are requirements for the particulars of claim in specific types of claim and situations. Here, the claim is based on a written agreement and so the written agreement should be attached. The claim is not based on an oral agreement or an agreement by conduct and so particulars of words spoken or conduct are not required. It is not the purpose of the particulars of claim to set out evidence (from an expert or a witness); this is provided at a later stage)

How well did you know this?
1
Not at all
2
3
4
5
Perfectly
36
Q

What is the purpose of the prayer in the particulars of claim?

A) A summary of the relief sought by the claimant

B) A summary of the duty, breach, causation and loss

C) A summary of the interest sought by the claimant

D) A summary of the parties

E) A summary of the damages sought by the claimant

A

A) A summary of the relief sought by the claimant

(Correct. The particulars of claim closes with a summary of the remedies sought by the claimant and so gives the court and the defendant a quick means of ascertaining what the claimant actually wants. It includes damages and, if claimed, interest)

How well did you know this?
1
Not at all
2
3
4
5
Perfectly
37
Q

You have just been instructed by a new client. Today’s date is 15th December and you advise your client that the limitation for its proposed claim expires on 27th December in 12 days’ time. The Practice Direction on Pre-action Conduct applies to the claim. Your client would like to proceed with the claim, but will not be available to assist with case preparation until early in January. Your client wants to know whether it is possible to postpone issuing proceedings until early in January? What advice should you give your client?

A) Your client should comply with the Practice Direction on Pre-action Conduct prior to issuing proceedings as failure to do so will result in adverse costs consequences.

B) Your client should write a letter to the defendant setting out the nature of the proposed claim and await a response as a demonstration of good conduct in the proceedings in accordance with the Practice Direction on Pre-action Conduct.

C) Your client should write a letter to the defendant setting out the nature of the proposed claim and suggesting that they agree to postpone proceedings until early in January.

D) Your client should issue proceedings straight away and then ask the defendant to agree to an application by your client to the court to stay proceedings so both parties can comply with the Practice Direction on Pre-action Conduct.

E) Your client should issue the claim form straight way but wait to serve it until the beginning of January when it is able to attach the particulars of claim to provide the defendant with more information on its underlying claim

A

D) Your client should issue proceedings straight away and then ask the defendant to agree to an application by your client to the court to stay proceedings so both parties can comply with the Practice Direction on Pre-action Conduct

(CORRECT: Given the tight timelines and the application of the Practice Direction on Pre-action Conduct the best course of action for the client is to issue the claim form straight away (so preventing the claim from being time-barred by limitation) but contact the defendant so that they can apply to the court asking for a stay to proceedings allowing compliance with the Practice Direction (paragraph 17). The other options set out do not fully address the duty owed by the claimant to comply with the Practice Direction or deal effectively with the risk of the claim being time-barred due to limitation)

How well did you know this?
1
Not at all
2
3
4
5
Perfectly
38
Q

A solicitor has been instructed on behalf of a supermarket chain to act in relation to a breach of contract claim against one of the supermarket’s suppliers. The supermarket chain is a private company. Having completed the requirements set out in the Practice Direction on Pre-action Conduct and having failed to settle the claim, the solicitor drafts the particulars of claim on behalf of their client. The managing director of the supermarket chain will sign the statement of truth on the particulars of claim. Which of the following gives the correct format for the statement of truth?

A) The Claimant believes that the facts stated in these particulars of claim are true. I understand that proceedings for contempt of court maybe brought against anyone who makes, or causes to be made, a false statement in a document verified by a statement of truth without an honest belief in its truth.

B) The Claimant understands that proceedings for contempt of court maybe brought against anyone who makes, or causes to be made, a false statement in a document verified by a statement of truth without an honest belief in its truth.

C) I am duly authorised by the Claimant to sign these particulars of claim. I understand that proceedings for contempt of court maybe brought against anyone who makes, or causes to be made, a false statement in a document verified by a statement of truth without an honest belief in its truth.

D) The Claimant believes that the facts stated in these particulars of claim are true. The Claimant understands that proceedings for contempt of court maybe brought against anyone who makes, or causes to be made, a false statement in a document verified by a statement of truth without an honest belief in its truth.

E) I believe that the facts stated in these particulars of claim are true. I understand that proceedings for contempt of court maybe brought against anyone who makes, or causes to be made, a false statement in a document verified by a statement of truth without an honest belief in its truth.

A

D) The Claimant believes that the facts stated in these particulars of claim are true. The Claimant understands that proceedings for contempt of court maybe brought against anyone who makes, or causes to be made, a false statement in a document verified by a statement of truth without an honest belief in its truth

(Well done. This is the correct form of a statement of truth where a statement of case (here, the particulars of claim) is being signed on behalf of a company. This was covered in the element ‘General rules in relation to statements of case’)

How well did you know this?
1
Not at all
2
3
4
5
Perfectly
39
Q

Your client wishes to commence proceedings for breach of contract against a supplier for defective goods. The amount of the claim is £108,000 (inclusive of £18,000 VAT). The claim is not complex. In which court can your client issue proceedings?

A) Because the case will be allocated to the multi-track, your client ought to commence proceedings in the High Court.

B) If your client commences proceedings in the County Court it is likely that the court will transfer it to the High Court.

C) Your client must issue proceedings in the High Court.

D) Your client has a choice as to whether to issue proceedings in the County Court or the High Court.

E) Your client must issue proceedings in the County Court given that the principal amount is £90,000.

A

D) Your client has a choice as to whether to issue proceedings in the County Court or the High Court

(Correct. Given that the amount is over £100,000 (even if this is only as a result of VAT), your client has a choice as to where to issue proceedings (7A PD 2.1 and CPR 16.3(6)). However, facts here may point towards the Count Court being the better choice given the complex is not complex and the value is only just over the £100,000 threshold)

How well did you know this?
1
Not at all
2
3
4
5
Perfectly
40
Q

A client is claiming damages for negligence by a solicitor. The solicitor works in a city law firm’s commercial property department and is listed in Chambers & Partners as a specialist in commercial property law. It is alleged that the solicitor negligently advised the client about the meaning of a restrictive covenant which has directly let to the client suffering loss. Which of the following statements best describes the standard of care that the solicitor should have exercised in this case?

A) The solicitor must act to the standard of a reasonably competent practitioner specialising in commercial property.

B) The solicitor must act to the standard of a reasonably competent practitioner.

C) The solicitor must act to the standard of a reasonably competent practitioner specialising in commercial property and recommended in Chambers & Partners.

D) The solicitor must act to the standard acceptable to a responsible body of practitioners.

E) The solicitor must act to the standard the majority of solicitors would act.

A

A) The solicitor must act to the standard of a reasonably competent practitioner specialising in commercial property

(The solicitor owes the client a duty to exercise reasonable care and skill (in both contract and tort). The duty owed is the same in contract and tort. The duty in contract derives from an implied term to act with reasonable care and skill pursuant to s.13 Supply of Goods and Services Act 1982. The same duty is owned in tort as Hedley Byrne v Heller (and later Caparo v Dickman) established that a person who makes a negligent statement could owe a duty of care to a person who suffered pure economic loss through reliance on that statement if there is a special relationship between the parties. The solicitor / client relationship is an established category of special relationship where it is reasonable to impose a duty. The standard is that of a reasonably competent solicitor. The court will consider that a solicitor is not normally negligent if acting in accordance with general practice (Bolam), but that is not conclusive and in deciding what is reasonable, the court may look at the level of skill it believes members of the professional ought to have had (Bolitho). There is a slightly higher standard for someone of greater experience and skill, for example, specialising in commercial property, in contrast to a solicitor who is a general practitioner, but no higher standard. The individual Chamber’s recommendation does not affect the standard. This question is a reminder that you need to have knowledge of contract and tort. Please remember that you have access to contract and tort materials on Adapt)

How well did you know this?
1
Not at all
2
3
4
5
Perfectly
41
Q

Your client is a company who is intending to commence proceedings against a partnership claiming professional negligence. The claim is for £500,000. As required, the company and the partnership have been following the Professional Negligence Pre-action Protocol. Recently, however, the solicitors for the potential defendant partnership were 2 days late in responding to the letter of claim without seeking an extension or offering any explanation. Which of the following statements best describes what your client should do, assuming it wishes to commence proceedings as soon as possible?

A) The company should expect more favourable case management orders since it has complied with the protocol and the partnership has not.

B) The company should commence proceedings as the Pre-action Protocol has not been followed by the partnership.

C) The company should raise the partnership’s non-compliance with the court and ask the court to impose costs sanctions for non-compliance with the protocol.

D) The company should ignore the fact that the partnership was 2 days late and carry on complying with the Pre-action Protocol.

E) The company should not raise the partnership’s non-compliance with the protocol at the moment but instead ask the court to make an adverse costs order at the end of the case (CPR 44.2) to reflect the non-compliance.

A

D) The company should ignore the fact that the partnership was 2 days late and carry on complying with the Pre-action Protocol

(CORRECT. Parties are expected to comply with the spirit of the protocols and the court will not be concerned with trivial breaches (paragraph 13 PD on Pre-action Conduct))

How well did you know this?
1
Not at all
2
3
4
5
Perfectly
42
Q

On receipt of the claimant’s claim form and particulars of claim, you advised your client, the defendant, that it has a good defence. You have just returned from holiday 3 days late due to your flight being cancelled. In your absence, judgment in default was entered against your client as you failed to file an acknowledgement of service or defence within 14 days of deemed service of the particulars of claim.

What is the best answer about whether there is anything you can do to enable your client to defend the claim?

A) Issue an application to set aside judgment as the judgment in default has been wrongly entered.

B) There is nothing that can usefully be done.

C) Issue an application to set aside judgment in default as the defendant has some real prospect of defending the claim and the application is made promptly.

D) Issue an application to strike out the claimant’s claim.

E) Issue an application to set aside judgment in default as the defendant is not at fault for failing to file acknowledgement of service in time.

A

C) Issue an application to set aside judgment in default as the defendant has some real prospect of defending the claim and the application is made promptly

(Correct. The application to set aside judgment in default should be made promptly and should show some merit to the defence. The other answers are incorrect because ‘fault’ is not necessarily relevant, the judgment has been correctly entered, and there is no basis for applying for strike-out)

How well did you know this?
1
Not at all
2
3
4
5
Perfectly
43
Q

A building company is suing a sub-contractor for breach of contract in the High Court. The claim is valued at £500,000 and it is a few days before the defence is due to be served. The sub-contractor needs to collate some financial information from its quantity surveyor about the works in order to finalise its defence. The quantity surveyor has just confirmed that it is unable to supply this information for at least the next 2 weeks. The defendant requests an extension of 3 weeks to serve its defence but the building company refuses to agree this extension of time. The sub-contractor therefore applies to the court for an extension of 3 weeks.

Which of the following arguments will be the most persuasive when the court is considering granting this extension?

A) Whether the date of the final hearing will be affected.

B) Whether it would be just and proportionate to grant the extension due to the problem with the quantity surveyor.

C) Whether the case is in the High Court

D) Whether the sub-contractor’s prospects of successfully defending the claim are good.

E) Whether the building company’s prospects of success in the claim are good.

A

B) Whether it would be just and proportionate to grant the extension due to the problem with the quantity surveyor

(Correct. When the court is exercising its case management powers, it will want to further the overriding objective to ensure that any extension enables the court to deal with the case justly and at proportionate cost. This will be alongside the reasons why an extension is required by the sub-contractor / not agreed by the building company)

How well did you know this?
1
Not at all
2
3
4
5
Perfectly
44
Q

On Thursday 1 October, your client issued a claim form in the High Court against the defendant to the claim claiming £125,000 being the price of goods sold and delivered. Unfortunately, the parties did not follow the Practice Direction on Pre-action Conduct. The claim form and particulars of claim were served together by first class post on Wednesday 14 October. The defendant has not yet filed any response to the claim. It is now Monday 2 November. Your client wishes to enter judgment in default for the sum of £125,000 plus interest. Which one of the following answers is correct?

A) Your client may NOT enter judgment in default because the deadline for filing an acknowledgment of service has not elapsed.

B) Your client may enter judgment in default because at least 14 days have elapsed since the particulars of claim were deemed to be served.

C) Your client may enter judgment in default because at least 14 days have elapsed since the claim form was issued.

D) Your client may NOT enter judgment in default because the defendant does not have to file an acknowledgment of service and the deadline for filing a defence has not elapsed.

E) Your client may NOT enter judgment in default because your client did not write a letter of claim in accordance with the Practice Direction on Pre-action Conduct.

A

B) Your client may enter judgment in default because at least 14 days have elapsed since the particulars of claim were deemed to be served

(Correct. Default judgment may be obtained in default of the defendant filing an acknowledgment of service and/or a defence (CPRs 12.1 and 12.3). The defendant should have filed an acknowledgement of service within 14 days of service of the claim form where the claim form and particulars of claim were served together or within 14 days after service of the particulars of claim where the claim form stated that the particulars of claim was to follow (CPR 10.3(1)(a)); or a defence within 14 days of service of the particulars of claim (CPR 15.4). The deemed date of service of the claim form and the particulars of claim was Friday 16 October and the defendant should have filed either an acknowledgment of service or a defence within 14 days after the deemed date of service which was Friday 30 October. This date has now passed and it is open for the claimant to apply for default judgment)

How well did you know this?
1
Not at all
2
3
4
5
Perfectly
45
Q

A firm is acting on behalf of a defendant in a litigation matter. The claimant served the defendant in England and Wales with the claim form and particulars of claim by personal service at 2.00 pm on Monday 5th July. The firm would like to serve the defence on the latest date possible without seeking permission from the claimant.

Based on the above information, the defendant, acting prudently, should file and serve its defence by when?

A) 30 August

B) 21 July

C) 2 August

D) 4 August

E) 19 July

A

C) 2 August

(Correct. The general rule is that the period for filing a defence is, a) 14 days after service of the particulars of claim; or b) If the defendant files an acknowledgement of service, 28 days after service of the particulars of claim (CPR 15.4). Time starts from the date of deemed service of the particulars of claim and the latest date to file a defence is 28 days after this date as long as an acknowledgement of service is filed.
If the particulars of claim are attached to or served with the claim form in a separate document and contained in the same envelope, then there is some ambiguity in the rules as to whether the rules for service of the CF (CPR 6.14) apply to the particulars of claim too, or whether the rules for documents other than the CF (CPR 6.26) apply. This could lead to the claim form and particulars of claim being deemed served on different dates, despite being in the same envelope.
A party responding to proceedings would be prudent to act on the assumption that the particulars are served on the earliest possible date. Therefore, here, deemed service of the particulars of claim (served within England and Wales personally before 4pm on a business day) is the same day, Monday 5 July (CPR 6.26). A period of time expressed as a number of days is computed as clear days which means the day on which the period begins, 5 July, is not included. The day on which the period ends is not included if it ends in an event (otherwise, it is included).
This means counting time begins on a clear day, 6 July, and ends on the twenty-eighth day of counting, here, 2 August, as the period does not end in an event ie a hearing)

How well did you know this?
1
Not at all
2
3
4
5
Perfectly
46
Q

The claimant is an investor bringing a claim against a former advisor, alleging negligent advice. Your client is the defendant advisor. The claimant alleges in paragraph 6 of its particulars of claim that ‘in breach of the implied term of the contract to exercise reasonable skill and care, the defendant failed to provide the claimant with adequate information from which to make an informed decision as to whether or not to proceed with the investment’. The advisor tells you that this is not true and that the claimant was provided with adequate information in a letter dated 6 April from which to make an informed decision. Your case analysis confirms that the information in the letter was adequate to discharge the advisor’s duty of care.

How should the defendant respond to paragraph 6 of the particulars of claim in the defence?

A) ‘The defendant requires proof of paragraph 6 of the particulars of claim.’

B) ‘The defendant denies paragraph 6 of the particulars of claim as the claimant was provided with adequate information with which to make an informed decision in a letter dated 6 April.’

C) ‘The defendant requires proof of paragraph 6 of the particulars of claim as the claimant was provided with adequate information with which to make an informed decision in a letter dated 6 April.’

D) ‘The defendant admits paragraph 6 of the particulars of claim.’

E) ‘The defendant denies paragraph 6 of the particulars of claim.’

A

B) ‘The defendant denies paragraph 6 of the particulars of claim as the claimant was provided with adequate information with which to make an informed decision in a letter dated 6 April.’

(Well done. As your client has another version of events, your client can deny the paragraph and set out that other version of events. In relation to the other answers, it would clearly be wrong to admit the paragraph, and putting the claimant to proof is only appropriate if the defendant can neither admit nor deny)

How well did you know this?
1
Not at all
2
3
4
5
Perfectly
47
Q

A counterclaim must…

A) Be based on a cause of action in which the defendant can establish duty, breach, causation and loss.

B) Always be heard with the main claim.

C) Be related to the claimant’s cause of action in the main claim.

D) Arise out of the same facts as the main claim.

E) Be made at the same time as the defendant files their defence.

A

A) Be based on a cause of action in which the defendant can establish duty, breach, causation and loss

(Correct. A counterclaim is a claim like any other and these four essential elements of the cause of action are essential)

How well did you know this?
1
Not at all
2
3
4
5
Perfectly
48
Q

A claimant claims £125,000 from the defendant in the main claim. The defendant makes a counterclaim of £50,000 and, as the facts underlying the counterclaim give rise to a set off, the defendant also raises this in defence to the main claim. Assuming both claimant and defendant win their respective claims, what will the final outcome be here?

A) The claimant will pay the defendant £75,000.

B) The defendant will pay the claimant £75,000.

C) The defendant will pay the claimant £175,000.

D) The defendant will pay the claimant £125,000.

E) The claimant will pay the defendant £50,000.

A

B) The defendant will pay the claimant £75,000

(Correct. The defendant will owe the claimant the £125,000 the claimant was awarded in their claim, less the £50,000 the defendant was awarded from the claimant in the counterclaim and which will be deducted by way of set off. The balance is £75,000)

How well did you know this?
1
Not at all
2
3
4
5
Perfectly
49
Q

A reply…

A) Is a compulsory statement of case for the claimant following the service of a defence and counterclaim.

B) Will be served by the claimant after the defence only if the claimant wishes to allege facts in answer to the defence which were not originally included in the claim.

C) Will usually be followed by further statements of case such as an amended defence.

D) Should be filed within 14 days after service of the defence.

E) Is a compulsory statement of case for the claimant following the service of a defence.

A

B) Will be served by the claimant after the defence only if the claimant wishes to allege facts in answer to the defence which were not originally included in the claim

(Correct. The reply is an optional statement of case and will only be served if needed. Please see CPR 15.8)

How well did you know this?
1
Not at all
2
3
4
5
Perfectly
50
Q

C is suing D1 and D2. D1 has the following claims: i) a counterclaim against C based on an unpaid invoice; and ii) a claim against D2 for a contribution or indemnity in respect of any damages D1 is ordered to pay to C.

What type of additional claim is D1’s claim against D2?

A) An additional claim for a contribution or indemnity against a third party

B) A counterclaim against a person other than the claimant

C) An additional claim for a contribution or indemnity against an existing party

D) An additional claim for a remedy other than a contribution or indemnity against an existing party

E) A classic counterclaim.

A

C) An additional claim for a contribution or indemnity against an existing party

(Correct. D1’s claim against D2 is a claim against an existing party for a contribution or indemnity (CPR 20.6))

How well did you know this?
1
Not at all
2
3
4
5
Perfectly
51
Q

C is suing D1 and D2. D1 brings an additional claim against E (CPR 20.7). In the proceedings E will be referred to as…

A) The Additional Claim Defendant.

B) The Part 20 Defendant.

C) The Fourth Party.

D) The Third Defendant.

E) The Third Party.

A

E) The Third Party.

(Correct. Additional parties will be referred to in accordance with the order in which they are joined to the proceedings (20 PD 7.4). E is the first additional party to join the proceedings so is the Third Party)

How well did you know this?
1
Not at all
2
3
4
5
Perfectly
52
Q

C is suing D. D brings an additional claim (CPR 20.7) against E. If E fails to respond to the additional claim within the applicable time limits…

A) D will not be able to apply for default judgment but E will be deemed to admit the additional claim. D will still need to prove any issues from the main claim between C and D in so far as they are relevant to the issues in the additional claim.

B) D will not be able to apply for default judgment but E will be deemed to admit the additional claim and will be bound by the judgment given at the trial of the main claim between C and D in so far as it is relevant to the issues in the additional claim.

C) D will be able to apply for default judgment against E in the additional claim and E will be bound by the judgment given at the trial of the main claim between C and D in so far as it is relevant to the issues in the additional claim.

D) E will be deemed to intend to defend the additional claim but will be bound by the judgment given at the trial of the main claim between C and D in so far as it is relevant to the issues in the additional claim.

E) D will be able to apply for summary judgment against E in the additional claim and E will be bound by the judgment given at the trial of the main claim between C and D in so far as it is relevant to the issues in the additional claim.

A

B) D will not be able to apply for default judgment but E will be deemed to admit the additional claim and will be bound by the judgment given at the trial of the main claim between C and D in so far as it is relevant to the issues in the additional claim.

(Correct. The additional claim will have been accompanied by an acknowledgment of service form and response pack and needs to be responded to in the usual way, failing which it will be deemed admitted as described in this option (CPR 20.11(2)))

How well did you know this?
1
Not at all
2
3
4
5
Perfectly
53
Q

C issues and serves a claim on D Limited following extensive dealings with the managing director of D Limited. D Limited defends the claim and the claim is now at the exchange of evidence stage. Although the claim was in time when issued, the limitation period has now expired. It transpires that the claim form mistakenly names the defendant as E Limited rather than D Limited.

Will C be able to amend the claim form to substitute E Limited with D Limited?

A) C will be permitted to amend the claim form but only if D Limited consents to this.

B) C will be permitted to amend the claim form changing E Limited to D Limited as this was a mistake in name only with no doubt as to the actual identity of the defendant.

C) C will not be permitted to amend the claim form as E Limited is a completely different entity and has a different identity from D Limited.

D) C will be permitted to amend the claim form as the limitation period was current when the original claim was issued.

E) C will not be permitted to amend the claim form as the limitation period for this has now expired and this would be a new claim against D Limited which is out of time.

A

B) C will be permitted to amend the claim form changing E Limited to D Limited as this was a mistake in name only with no doubt as to the actual identity of the defendant

(Correct. This situation falls within CPR 17.4(3). D Limited was always the intended defendant and the mistake causes no reasonable doubt as to the identity of the defendant)

How well did you know this?
1
Not at all
2
3
4
5
Perfectly
54
Q

Permission to amend the particulars of claim to add a party after the claim form has been served but before the limitation period has expired should be given if…

A) The claimant has a reasonable prospect of success in the claim against the new party.

B) The amendment is agreed by all other parties.

C) The amendment is made not too close to the trial date.

D) The amendment is ‘necessary’.

E) The amendment is ‘desirable’.

A

E) The amendment is ‘desirable’.

(Correct. This is the main test as set out in CPR 19.2)

How well did you know this?
1
Not at all
2
3
4
5
Perfectly
55
Q

Amended statements of case must…

A) Use coloured amendments.

B) Use margin references to indicate where amendments have been made.

C) Show the original text as well as the new text.

D) Be verified by a statement of truth unless the court orders otherwise.

E) Use a numerical code for amendments.

A

D) Be verified by a statement of truth unless the court orders otherwise

(Correct. See CPR 22.1(2))

How well did you know this?
1
Not at all
2
3
4
5
Perfectly
56
Q

When is the court most likely to make an order for further information?

A) After the party seeking the information has made the request to the other party on the telephone.

B) After a voluntary request to the other party has been refused or not responded to.

C) After the parties have reached an agreement about the request.

D) After the parties have indicated that no further information is required.

E) After the parties have complied with the overriding objective.

A

B) After a voluntary request to the other party has been refused or not responded to

(It is correct to say that the court is most likely to make an order for further information after a voluntary request to the other party has been refused or not responded to. The court expects the parties make requests on a voluntary basis first and the parties should only make an application to the court for an order under CPR 18.1 if the request cannot be resolved – see 18 PD 1.
The other answers are incorrect:
After the parties have complied with the overriding objective. Parties are expected to comply with the overriding objective at all times and it will always be a factor that the court takes into account when making an order, but it is not the ‘reason’ for the court making an order here.
After the parties have reached agreement about the request. If the parties have reached agreement, they do not need a CPR 18.1 court order – 18 PD 1.
After the party seeking the information has made the request to the other party on the telephone. The party seeking information should serve a written request on the other party stating a date for a response (and allowing the other party a reasonable amount of time to respond) – 18 PD 1.
After the parties have indicated that no further information is required. If no further information is required by either party, they do not need a CPR 18.1 court order – 18 PD 1)

How well did you know this?
1
Not at all
2
3
4
5
Perfectly
57
Q

When an application for an order for further information is made to the court, it should be made as…

A) The use of the court’s general case management powers – CPR 3.1

B) Relief from sanctions – CPR 3.9

C) An amendment to a statement of case – CPR 17

D) An interim application – CPR 23

E) The court’s allocation to track – CPR 26

A

D) An interim application – CPR 23

(The correct answer is that an application for an order for further information is made as an interim application pursuant to CPR 23. The application should be made as an interim application under CPR 23 (18 PD 5).
The other options were incorrect:
The use of the court’s general case management powers – CPR 3.1
Relief from sanctions – CPR 3.9
The court’s allocation to track – CPR 26
An amendment to a statement of case – CPR 17
These relate to other types of application)

How well did you know this?
1
Not at all
2
3
4
5
Perfectly
58
Q

A party can obtain further information from the other party to clarify or give additional information in relation to any matter…

A) Which is in dispute in the proceedings

B) Which is contained in or referred to in a defence.

C) Which is contained in or referred to in a statement of case.

D) Which is contained in or referred to in a witness statement

E) Which is contained in or referred to in a particulars of claim

E) Which is contained in or referred to in a particulars of claim.
Correct

A

A) Which is in dispute in the proceedings

(The correct answer is that a party can obtain further information from the other party to clarify or give additional information in relation to any matter which is in dispute in the proceedings as explained at the beginning of this element.
The incorrect answers are:
Which is contained in or referred to in a statement of case.
Which is contained in or referred to in a defence.
Which is contained in or referred to in a particulars of claim.
Which is contained in or referred to in a witness statement.
The definition is broader than these options. The request is often in relation to a matter contained or referred to in a statement of case, but the court’s power is not limited to matters contained or referred to in a statement of case (or to any other particular document))

How well did you know this?
1
Not at all
2
3
4
5
Perfectly
59
Q

In a case involving four defendants, one of the defendants amends its defence to allege that the claimant was contributorily negligent. The court grants permission for this amendment. Which party will usually pay the costs of and arising from the amendment?

The unsuccessful party at the end of the trial in accordance with the general rule on costs (CPR 44.2).

A) The claimant

B) The defendant applying for the amendment

C) All the parties to the action

D) All the defendants

A

B) The defendant applying for the amendment

(Well done. This reflects the rule that a party applying for an amendment will usually be responsible for the costs of and arising from the amendment (PD17 and PD19). So, in this case it is likely that the defendant making the amendment would be responsible for the costs. Note that the rule is not obligatory, and the court can depart from it. Given this rule, it is unlikely that the costs would fall to be determined at the end of the trial as most other costs would (CPR 44.2))

How well did you know this?
1
Not at all
2
3
4
5
Perfectly
60
Q

Your client has received a request for further information by letter in respect of a defence it filed to a claim against it for misrepresentation. The defence as currently drafted fails to fully detail the oral representations made by your client before the contract was entered into. Your client agrees to provide the information requested. Which of the following most accurately describes the form your client’s response to the request for further information should take?

A) The client’s response should be in writing, signed, dated and be verified by a statement of truth.

B) The client’s response should be in the form of a letter in reply which will be signed and dated as normal.

C) The client’s response should take the form of an amended defence.

D) The client’s response should be in writing, signed and dated but need not be verified by a statement of truth.

E) The client should provide the information in whichever form is most proportionate including verbally, for example, in a telephone call.

A

A) The client’s response should be in writing, signed, dated and be verified by a statement of truth

(Correct. Whatever form the request has taken, the response must be in writing, dated and signed by either the party or their legal representative and also verified by a statement of truth. It could be by letter or by way of a formal reply. (PD 18))

How well did you know this?
1
Not at all
2
3
4
5
Perfectly
61
Q

The claimant claims £32,000. The defendant pleads £8,000 ‘set off’ in its defence and counterclaim against the claimant. If the claimant is successful at trial and the defendant is also successful at trial in its counterclaim, what is the effect of the judgment?

A) Defendant pays the claimant £24,000

B) Defendant pays the claimant £8,000

C) Claimant pays the defendant £8,000

D) Defendant pays the claimant £32,000

E) Defendant pays the claimant nothing

A

A) Defendant pays the claimant £24,000

(Correct. The set off applies and extinguishes £8,000 of the claimant’s judgment against the defendant. The defendant therefore pays the claimant the net sum of £24,000)

How well did you know this?
1
Not at all
2
3
4
5
Perfectly
62
Q

A construction company intends to sue one of its sub-contractors, an electrician, for breach of contract. The claim form has been issued, but it has not yet been served. Before service of the claim form, the construction company decides to add a new defendant, a plumber, to the claim as an additional party. Which of the following best describes the position of the construction company?

A) The construction company will require the written consent of both the electrician and the plumber to add the additional party.

B) The construction company should seek the written consent of the electrician to add the additional party.

C) The construction company should apply to the court for permission to add the additional party.

D) The construction company should seek the written consent of the plumber to be added as an additional party.

E) The construction company will not require the court’s permission to add the additional party.

A

E) The construction company will not require the court’s permission to add the additional party.

(Well done. The claim form has been issued but not served. In this case, the court’s permission is not necessary to add the plumber as an additional party (nor is permission needed from either the electrician or the plumber) (CPR 19.4(1)))

How well did you know this?
1
Not at all
2
3
4
5
Perfectly
63
Q

During her employment as a delivery driver, your client, Sue, was involved in a road traffic accident with another car. Your client has sued the driver of the other car, Amir, for personal injuries. Amir, in turn, has issued a counterclaim against your client claiming damages for personal injuries that Amir also sustained in the road traffic accident. Amir wishes to use the same arguments in the counterclaim against your client’s employer. The employer is vicariously liable for your client’s actions. Can Amir add your client’s employer as a party to the proceedings?

A) No, as there is no Part 20 claim that provides for this situation.

B) Yes, as a counterclaim against a person other than the claimant (CPR 20.5)

C) Yes, as a counterclaim against the claimant (CPR 20.4)

D) Yes, as the defendant’s additional claim for contribution or indemnity from another party (CPR 20.6)

E) Yes, as an ‘other’ additional claim (CPR 20.7)

A

B) Yes, as a counterclaim against a person other than the claimant (CPR 20.5)

(Correct. Your client, Sue, is the claimant and Amir is the defendant. Amir is seeking to include a counterclaim against a person ‘other than the claimant’ (i.e. other than Sue, namely, the employer) where Sue and the employer are jointly involved in the situation. (CPR 20.5))

How well did you know this?
1
Not at all
2
3
4
5
Perfectly
64
Q

If an applicant makes a standard with notice interim application, what is the timeframe within which the applicant must serve a copy of the application notice and supporting documentation on the respondent?

A) As soon as practicable before the hearing

B) Not less than 3 clear days before the hearing

C) As soon as practicable but at least 3 clear days after issuing the application notice.

D) As soon as practicable but not less than 3 clear days before the hearing.

E) Not less than 14 clear days before the hearing

A

D) As soon as practicable but not less than 3 clear days before the hearing

(Correct: if an applicant makes a standard with notice interim application, they should serve the respondent a copy of the application notice and supporting documentation as soon as practicable but not less than 3 clear days before the hearing (CPR 23.7))

How well did you know this?
1
Not at all
2
3
4
5
Perfectly
65
Q

The directions order confirms that witness statements are due to be exchanged in 5 days’ time. The applicant wishes to apply to the court for extra time to exchange witness statements. The parties have been unable to agree an extension of time. How would the applicant make an interim application to apply for an extension of time to exchange witness statements?

A) Without notice as the object of the order would be defeated by providing notice.

B) With notice by filing at court an application notice and a witness statement and paying a fee.

C) With notice by filing at court an application notice, a witness statement and a draft order and paying a fee.

D) With notice by filing at court an application notice endorsed with a hearing date, a witness statement, a draft order and paying a fee.

E) Without notice due to the urgency

A

C) With notice by filing at court an application notice, a witness statement and a draft order and paying a fee

(Correct: an interim application is made by filing at court an application notice, a witness statement and a draft order and paying a fee)

How well did you know this?
1
Not at all
2
3
4
5
Perfectly
66
Q

The directions order confirms that witness statements are due to be exchanged in 5 days’ time. The applicant wishes to apply to the court for extra time to exchange witness statements. The parties have been unable to agree an extension of time. How would the applicant make an interim application to apply for an extension of time to exchange witness statements?

A) Without notice as the object of the order would be defeated by providing notice.

B) With notice by filing at court an application notice and a witness statement and paying a fee.

C) With notice by filing at court an application notice, a witness statement and a draft order and paying a fee.

D) With notice by filing at court an application notice endorsed with a hearing date, a witness statement, a draft order and paying a fee.

E) Without notice due to the urgency

A

C) With notice by filing at court an application notice, a witness statement and a draft order and paying a fee

(Correct: an interim application is made by filing at court an application notice, a witness statement and a draft order and paying a fee)

How well did you know this?
1
Not at all
2
3
4
5
Perfectly
67
Q

How, if at all, will a respondent know what evidence has been put forward by the applicant in support of a without-notice application?

A) The evidence will be served on the respondent after the hearing

B) The respondent will not know what evidence has been put forward in support of a without-notice application

C) The evidence will be served on the respondent before the hearing

A

A) The evidence will be served on the respondent after the hearing

(Correct. The applicant must serve the application notice, evidence in support and order on the respondent as soon as possible after the hearing (CPR 23.9))

How well did you know this?
1
Not at all
2
3
4
5
Perfectly
68
Q

The claimant files a claim form and particulars of claim. The defendant files an acknowledgment of service indicating an intention to defend the claim. The claimant applies for summary judgment. Both parties attend the summary judgment hearing where the judge dismissed the application and makes a costs order. What order is the court most likely to make next?

A) Conditional order

B) Directions order for the defendant to file and serve its defence

C) Order the parties to file a statement of costs

D) Allocation to a track

E) Order to set aside the summary judgment hearing

A

B) Directions order for the defendant to file and serve its defence

(A directions order for the defendant to file and serve its defence is correct. The summary judgment application was made before the defendant filed its defence and therefore the time for the defendant to file its defence is extended until after the summary judgment hearing (CPR 24.4(2)). The court has made a decision on summary judgment for dismissal of the application and so the claim will continue. The court may give directions for the defendant to file and serve the defence as this has not already taken place. An order to set aside the summary judgment hearing is incorrect as there is nothing on the facts to suggest this is appropriate e.g. both parties attended the summary judgment hearing. A conditional order is incorrect as this type of order is an alternative outcome to the outcome already arrived at (dismissal of the application) – not an additional order. Allocation to a track is incorrect. Whilst the court will delay allocating the matter to a track until after the summary judgment hearing, it will want to see the defendant’s case (in the defence) first. Order the parties to file a statement of costs is incorrect. This would have happened 24 hours before the hearing (CPR 44 PD 9.5) rather than after the summary judgment hearing to enable the court to make an order for costs at the hearing)

How well did you know this?
1
Not at all
2
3
4
5
Perfectly
69
Q

When is the first opportunity to a claimant can apply for summary judgment against the defendant without needing the court’s permission?

A) After the claimant has filed particulars of claim.

B) Any time after proceedings have commenced.

C) After the defendant has filed an acknowledgment of service.

D) At the same time as filing the directions questionnaire.

E) After the defendant has filed an acknowledgment of service or a defence.

A

E) After the defendant has filed an acknowledgment of service or a defence

(The correct answer is the claimant can apply for summary judgment ‘after the defendant has filed an acknowledgment of service or a defence’ (CPR 24.4). To apply before this, the court’s permission would be needed. An incorrect answer is ‘after the defendant has filed an acknowledgment of service’ as this is not the best answer; the defendant could choose to only file a defence which would then be the trigger for the first opportunity a claimant has to apply for summary judgment. An incorrect answer is ‘after the claimant has filed particulars of claim’ as the trigger is a step by the defendant, not the defendant. An incorrect answer is ‘any time after proceedings have commenced’; this is the rule for when the defendant can first apply for summary judgment, not the claimant. An incorrect answer is ‘at the same time as filing the directions questionnaire’; this is often an ideal time for either party to apply for summary judgment, but it is not the first opportunity)

How well did you know this?
1
Not at all
2
3
4
5
Perfectly
70
Q

The claimant ordered 20,000 laptops from the defendant. The laptops malfunction. The claimant issues a claim against the defendant claiming that 20,000 laptops are faulty. The defendant files a defence. The claimant believes that the defendant is entirely at fault and applies for summary judgment. If the claimant’s summary judgment application is successful, which order is most likely to be made by the court?

A) Dismissal of the application

B) Conditional order

C) Dismissal of the claim

D) Judgment on the claim

E) Judgment on part of the claim

A

D) Judgment on the claim

(The correct answer is judgment on the claim. The claimant’s claim is in relation to 20,000 laptops (the entire order) and so, if successful on summary judgment, the claimant will get judgment on the claim and the entire claim is over. Dismissal of the claim is incorrect as this would be appropriate where the defendant successfully applies for summary judgment; it is the claimant who has applied here. Dismissal of the application is incorrect as this is appropriate where either the claimant or defendant’s application for summary judgment has failed, the summary judgment application is dismissed and the claim continues to trial. Conditional order is incorrect as there is nothing on the facts to suggest that the judge is not wholly convinced of the merits of one party’s case necessitating conditions ordered by the court. Judgment on part of the claim is incorrect because there are problems with the entire order of 20,000 laptops. There is nothing on the facts yet to suggest that part of the order is not faulty)

How well did you know this?
1
Not at all
2
3
4
5
Perfectly
71
Q

The court must not make an interim payment of more than________________________ .

Choose the correct missing words to complete the sentence.

A) A reasonable proportion of the estimated costs

B) A reasonable proportion of the likely amount of the final judgment

C) 66% of the likely amount of the final judgment

D) A reasonable proportion of the sum claimed by the claimant

A

B) A reasonable proportion of the likely amount of the final judgment

(Correct. See CPR 25.7)

How well did you know this?
1
Not at all
2
3
4
5
Perfectly
72
Q

Which of the following IS NOT one of the conditions on which an application for an interim payment can be based?

A) The defendant has admitted liability to pay damages (or some other sum of money) to the claimant.

B) It is satisfied that, if the claim went to trial, the claimant would obtain judgment for a substantial amount of money against the defendant from whom he is seeking an order for an interim payment.

C) The defendant has no real prospect of successful defending the claim and there is no other compelling reason why the claim should be disposed of at trial.

D) The claimant has obtained judgment against that defendant for damages to be assessed (or for a sum of money other than costs) to be assessed

A

C) The defendant has no real prospect of successful defending the claim and there is no other compelling reason why the claim should be disposed of at trial

(Correct. This IS NOT one of the conditions on which an application for an interim payment can be based. It is part of the rules relating to summary judgment. All the other answers are conditions on which an application for an interim payment can be based)

How well did you know this?
1
Not at all
2
3
4
5
Perfectly
73
Q

You act for a claimant in a personal injury claim. The claimant was badly injured by the defendant in a road traffic accident several months ago. The defendant has admitted liability in its defence. The claim is now at the case management stage. The claimant has not yet been able to return to work due to injuries sustained in the accident and has used all their savings to live on since the accident. The claimant has to move into new rented accommodation and needs money for the deposit and general expenditure for food and travel. Can the claimant apply to the court for an interim payment from the defendant at this stage of the proceedings?

A) Yes, because the period for filing an acknowledgment of service has expired.

B) Yes, because the period for filing a defence has expired.

C) No, because a final judgment has not been made against the defendant.

D) No, because the claimant can only claim an interim payment pre-action.

E) No, because the claimant cannot claim an interim payment during proceedings.

A

A) Yes, because the period for filing an acknowledgment of service has expired

(Correct. The claimant can apply to the court for an interim payment from the defendant because the period for filing an acknowledgment of service has expired an application for an interim payment can be made after this stage of the proceedings has been reached (CPR 25.6).
The incorrect answers are: Yes, the period for filing a defence has expired; this is not the rule (CPR 25.6). No, because the claimant cannot claim an interim payment during proceedings; the intention of interim payments is to allow a claimant to seek financial assistance on account of damages in certain situations during proceedings (CPR 25.6). No, because the claimant cannot claim an interim payment pre-action; whilst the claimant can agree an interim payment with the defendant pre-action, it cannot make a court application for an interim payment at that stage. No, because a final judgment has not been made against the defendant; whilst this is one of the conditions for applying for an interim payment, it is not the only condition and if another of the prescribed conditions is met, this would not preclude the claimant from applying (provided the claimant also meets the relevant timing requirements)(CPR 25.6)

How well did you know this?
1
Not at all
2
3
4
5
Perfectly
74
Q

The claimant has made a personal injury claim against the defendant. The court has ordered judgment on liability in favour of the claimant, but the amount of damages have yet to be assessed. The claimant would like an interim payment as soon as possible and so asks the defendant for an interim payment. What is the best course of action for the claimant if the defendant does not agree to voluntarily make an interim payment?

A) Apply to the court for an interim payment as the defendant has admitted liability.

B) Do not apply to the court for an interim payment and wait until damages are assessed.

C) Apply to the court for an interim payment as the claimant is entitled to an interim payment in a personal injury claim.

D) Apply to the court for an interim payment as the claimant has obtained judgment against the defendant.

E) Apply to the court for an interim payment as if the claim went to trial the claimant would obtain judgment for a substantial amount of money against the defendant.

A

D) Apply to the court for an interim payment as the claimant has obtained judgment against the defendant.

(Correct – the best course of action for the claimant to take if the defendant does not agree to voluntarily make an interim payment is to apply to the court. The claimant must satisfy one of the prescribed conditions (CPR 25.7) and, on the facts, has done this as the claimant has obtained judgment against the defendant.
The other options were not the best course of action: • The claimant cannot apply to the court for an interim payment as, on the facts, the defendant has not admitted liability. • The claimant cannot apply to the court for an interim payment as you are not given any facts to suggest that if the claim went to trial the claimant would obtain judgment for a substantial amount of money against the defendant. • It is incorrect to say that a claimant is entitled to an interim payment in a personal injury claim. One of the prescribed conditions (CPR 25.7) must be satisfied in any type of claim. • It is not the best course of action to say ‘do not apply to the court for an interim payment and wait until damages are assessed’ because if one of the prescribed conditions (CPR 25.7) applies, the claimant has a choice to apply for an interim payment)

How well did you know this?
1
Not at all
2
3
4
5
Perfectly
75
Q

The defendant must satisfy the court of two matters before an order for security for costs can be made. One or more of the prescribed conditions in the rules must be satisfied, and having regard to all the circumstances, it must be ________ to make an order.

Choose the correct missing word to complete the sentence.

A) Proportionate

BJ Necessary

C) Reasonable

D) Just

A

D) Just

(Correct. See CPR 25.13. It would be wrong to say that it must be ‘necessary’. Whether it is ‘reasonable’ and ‘proportionate’ will be relevant considerations, and part of considering what is ‘just’, but ‘just’ is the key concept)

How well did you know this?
1
Not at all
2
3
4
5
Perfectly
76
Q

The claimant company is a breakfast cereal manufacturing company. The claimant has issued proceedings to claim losses of £200,000 from the defendant, a grain supplier, for breach of contract. You act for the defendant who has submitted its defence. Yesterday, a leading food trade magazine published an article about the claimant’s inability to meet customer demand for breakfast cereal and that it is experiencing cashflow difficulties. The defendant has therefore today checked the claimant’s most recent annual accounts filed at Companies House which show a loss of £215,000.

What would be the most appropriate interim application that the defendant might make?

A) Extension of time to submit its defence

B) Interim payment

C) Security for costs

D) Interim prohibitory injunction

E) Interim mandatory injunction

A

C) Security for costs

(Correct – security for costs – because, if successful, a security for costs order would require the claimant to pay a sum of money into court (or by some other manner) to protect the defendant should it successfully defend the claimant’s case and be awarded costs. The defendant wants this protection as it has reason to believe that the claimant will be unable to pay the defendant’s costs if ordered to do so.
The other options were incorrect: Interim payment is incorrect because that is an application made by a claimant (for an interim payment on account of damages which the defendant may be held liable to pay). Interim prohibitory injunction is incorrect because the defendant does not need an order from the court to require the claimant to refrain from doing an act which is causing irreparable or immeasurable damage to the defendant . Interim mandatory injunction is incorrect because the defendant does not need an order from the court to require the claimant to do an act to avoid causing irreparable or immeasurable damage to the defendant. Extension of time to submit its defence (CPR 3.1) is incorrect because the defendant has already filed its defence and so does not need extra time to do so)

How well did you know this?
1
Not at all
2
3
4
5
Perfectly
77
Q

The claimant to a claim proceeding in the courts of England and Wales is a company with its registered office and central administration in North Carolina, USA. It has not got offices or assets anywhere outside of North Carolina, USA. You do not yet have access to its accounts in order to assess its financial situation. The defendant is a company registered in England and Wales which is currently in financial difficulties.

On the available facts, which condition, if any, could the defendant rely on to make an interim application for security for costs assuming that it is also able to satisfy the court that, in all the circumstances, it is just to make the order?

A) The defendant could rely on the condition that the claimant has taken steps in relation to its assets that would make it difficult to enforce an order for costs against it.

B) The defendant cannot rely on any of the conditions.

C) The defendant could rely on the condition that the claimant is a company and there is reason to believe it will be unable to pay the defendant’s costs if ordered to do so.

D) The defendant could rely on the condition that the claimant is resident out of the jurisdiction.

E) The defendant could rely on the condition that the defendant is an impecunious company.

A

D) The defendant could rely on the condition that the claimant is resident out of the jurisdiction

(Correct – the defendant could rely on the condition that the claimant is resident out of the jurisdiction as it is in North Carolina, USA which is not a State bound by the 2005 Hague Convention (CPR 25.13(2)(a)). The other options were incorrect: The defendant could not rely on the condition that the claimant is a company and there is reason to believe it will be unable to pay the defendant’s costs if ordered to do so because although the claimant is a company, there is no evidence on the available facts that it is impecunious. The defendant could not rely on the condition that the claimant has taken steps in relation to its assets that would make it difficult to enforce an order for costs against it because there is nothing on the facts to suggest that the claimant has taken any steps in relation to its assets. It cannot be said that the defendant cannot rely on any of the conditions because one does apply. The defendant could not rely on the condition that the defendant is an impecunious company because this is not a condition: all conditions are based on the claimant’s status/actions, not the defendant’s)

78
Q

The claimant and defendant are in dispute over which of them is the legal and beneficial owner of the voting shares in a company, Levison Limited (‘Levison’). Last week, the claimant obtained an interim injunction without notice, which prohibits the defendant from exercising the votes attached to these shares at a Levison shareholder meeting which is taking place next week. The claimant’s main argument for the injunction is that the defendant was intending to vote at the meeting in order to push through a resolution that will benefit the defendant’s expansion plans, but which would disadvantage the claimant. The defendant’s position is that, as the claimant actually owns a majority share in Levison aside from the disputed shares, the defendant’s vote would not have affected the outcome of the shareholder meeting. The with notice hearing is scheduled for tomorrow morning. Assuming the defendant’s statement is correct, which of the following outcomes is most likely at the with notice hearing?

A) The injunction will continue because the defendant should still not be exercising the voting rights attached to the disputed shares.

B) The injunction will be discharged and the court will accept an undertaking by the defendant not to do the act.

C) The injunction will be discharged at the with notice hearing, without any undertaking being required from the defendant not to do the act.

D) The injunction will be discharged at the eventual trial.

E) The injunction will continue because injunctions granted without notice are always expressed to last until trial.

A

C) The injunction will be discharged at the with notice hearing, without any undertaking being required from the defendant not to do the act

(Correct: if the defendant’s statement is correct, then the injunction will most likely be discharged at the with notice hearing. The principle that an injunction will not be obtained when it would serve no practical purpose (‘equity does not act in vain’) would most likely mean that the injunction would be discharged at the with notice hearing and the court would also consider whether or not to award damages to the defendant pursuant to the claimant’s undertaking in damages.
If the defendant’s statement is correct, the other options are less likely. The court is less likely to discharge the injunction at the eventual trial as there is evidence to suggest the injunction is not required now (equity does not act in vain) and the court will choose the course that carries the lower risk of injustice when exercising the balance of convenience; the trial may be some time away. The court is less likely to maintain the injunction on the basis that the defendant should still not be exercising the voting rights attached to the disputed shares as there is evidence to suggest the injunction is not required now (equity does not act in vain) and the court will choose the course that carries the lower risk of injustice when exercising the balance of convenience. This court is less likely to maintain the injunction on the basis that injunctions granted without notice are always expressed to last until trial as this reasoning is incorrect; in fact, injunctions made without notice will almost always last until the with notice hearing (rather than trial), at which point the court will decide whether they should continue. The court is less likely to discharge the injunction and accept an undertaking by the defendant not to do the act; on the facts the defendant has not offered an undertaking (and still intends to vote) and the court cannot force a party to offer an undertaking)

79
Q

Your client is concerned that one of its competitors is about to launch a new product onto the market in infringement of your client’s patents and copyrights. Your client wishes to start civil proceedings for infringement and would like to apply for an interim injunction against the competitor to prevent the new product being launched. At the injunction hearing, if damages would not be an adequate remedy for your client, the applicant, what would the court consider next?

A) The court will next consider whether, if the injunction is granted, the respondent would be adequately compensated under the applicant’s undertaking as to damages.

B) The court will next consider whether there is a serious question to be tried.

C) The court will next consider whether the defence has a real prospect of success.

D) The court will next consider where the balance of convenience lies. Please revisit the American Cyamamid guidelines in this element.

E) The court will next consider the merits of the case.

A

A) The court will next consider whether, if the injunction is granted, the respondent would be adequately compensated under the applicant’s undertaking as to damages

(It is correct that the court will next consider whether, if the injunction is granted, the respondent would be adequately compensated under the applicant’s undertaking as to damages. This is usually part of ‘step 2’ and if damages from the applicant’s undertaking would be an adequate remedy to the respondent, and if the applicant is financially able to satisfy any such undertaking, there are good reasons to grant an interim prohibitory injunction. Although the court may consider the other options at the injunction hearing, it is unlikely that they will be considered ‘next’ pursuant to American Cyanamid: The court will consider whether there is a serious question to be tried – the court would have already decided this step (step 1) before considering if damages are an adequate remedy because if this step cannot be met, the injunction will generally be refused. The court will consider where the balance of convenience lies – the court considers this step (step 3) if it cannot resolve an injunction application by finding the answer from step 2 (and, even though steps 2 and 3 are sometimes blurred, it would still be necessary for the court to consider whether the respondent would be adequately compensated before it can make any decision on the balance of convenience). The court will not separately consider whether the defence has a real prospect of success or the merits of the case at this stage: the court will consider whether there is a serious question to be tried as ‘step 1’, but after that, the key considerations are the adequacy of damages as set out above)

80
Q

Which of the following statements is correct in relation to the applicant’s cross-undertaking in damages in an application for an interim injunction?

A) The applicant makes the cross-undertaking in damages to the respondent

B) The court can order an applicant to make a cross-undertaking in damages

C) The grant of an interim injunction is often conditional on the applicant providing a cross-undertaking in damages.

D) The applicant’s cross-undertaking in damages is considered alongside whether there is a serious question to be tried

E) The applicant’s cross-undertaking in damages can only be for the protection of the respondent

A

C) The grant of an interim injunction is often conditional on the applicant providing a cross-undertaking in damages

(Correct – the grant of an interim injunction is often conditional on the applicant providing a cross-undertaking in damages. The other options are incorrect: The applicant makes the cross-undertaking in damages to the respondent – no, it makes the cross-undertaking to the court. The court can order an applicant to make a cross-undertaking in damages - no, the court cannot order any party to make an undertaking so here it is subject to the willingness of the applicant. The applicant’s cross-undertaking in damages can only be for the protection of the respondent – no, the cross-undertaking can be for the benefit of others that have suffered damage by reason of the injunction if it is subsequently held that the applicant ought not to have been granted an interim injunction. It depends on what the court requires. The applicant’s cross-undertaking in damages is considered alongside whether there is a serious question to be tried – no, the applicant’s cross-undertaking is usually considered as part of step 2 and sometimes 3 of the American Cyanamid. Serious question to be tried is step 1)

81
Q

An applicant makes an application for security for costs with a return date of Friday 22 October. What is the latest date that the applicant might be permitted to serve a copy of the application notice and supporting documentation on the respondent (without further order from the court)?

A) As soon as possible before the hearing

B) Friday, 15 October

C) Tuesday, 19 October

D) Thursday, 21 October

E) Monday, 18 October.

A

E) Monday, 18 October.

(Correct: if an applicant makes a with notice interim application, they should serve the respondent a copy of the application notice and supporting documentation as soon as possible but not less than 3 clear days before the hearing (CPR 23.7). The other options were not correct: • As soon as possible before the hearing – this is only part of the rule on the service time limit in this situation (CPR 23.7). • Tuesday, 19 October. This is not 3 ‘clear’ days before the hearing. • Friday, 15 October. This is not the correct time limit for standard with notice interim applications (CPR 23.7). • Thursday, 21 October (you could be confusing the application notice and evidence with the statement of costs, which can be served 24 hours before the hearing).)

82
Q

A claimant has commenced proceedings seeking in the region of £200,000 in relation to personal injuries. A defence has been filed defending the claim in full. It is clear that the claimant will be awarded at least £50,000 in relation to the injuries at trial, but the prospects of recovering more than this are unclear. The claimant needs to make adaptations to their house to cope with their injuries, but does not have the money to do so. Does an application for an interim payment by the claimant have good prospects of success?

A) Yes, because the claimant does not have any money to make the necessary adaptations.

B) No, because the defendant has not admitted liability.

C) Yes, because if the claim went to trial the claimant would obtain judgment for a substantial amount of money.

D) No, because the prospects of recovering the majority of the claim are uncertain.

E) No, because the claimant has not obtained judgment against the defendant.

A

C) Yes, because if the claim went to trial the claimant would obtain judgment for a substantial amount of money.

(Correct. CPR 25.7(1) provides that a condition giving the court jurisdiction to award an interim payment is that the court “is satisfied that, if the claim went to trial, the claimant would obtain judgment for a substantial amount of money (other than costs) against the defendant”, and this is clearly satisfied if ‘it is clear that the claimant will be awarded at least £50,000’ at trial. An admission of liability or having already obtained judgment are alternative conditions, but they are not necessary ones, so the absence of these does not prevent an application for an interim payment. It does not matter that the very good prospects only relate to 25% of the sum claimed, as long as this is still a substantial amount (but note that the court must not order an interim payment of more than a reasonable proportion of the likely amount of the final judgment – CPR 25.7(4)). The fact that the claimant does not have any money to make the necessary adaptations is not the main factor giving the application good prospects of success)

83
Q

For a summary judgment application, what is the latest date by which the applicant must serve the application documentation on the respondent if the hearing is to take place on Friday 22 October?

A) Friday 15 October

B) Monday 18 October

C) Thursday 7 October

D) Thursday 14 October

E) Friday 8 October

A

C) Thursday 7 October

(Correct – the applicant must give at least 14 days’ notice of the date fixed for the hearing (CPR 24.4(3)))

84
Q

A defendant makes an application for security for costs against the claimant two weeks before the trial of the claim. The claimant opposes the application.

The claimant company is based overseas in the USA with no assets or operations elsewhere. It has been financially solvent for many years holding both liquid and illiquid assets (as publicly available accounts across the years have shown), and has a strong claim against the defendant.

What is the most accurate description of the likely outcome of the defendant’s security for costs application?

A) The defendant is likely to be unsuccessful in its application because of its delay in making the application.

B) The defendant is likely to be successful in its application because the claimant is resident out of the jurisdiction (but is not resident in a state bound by the Hague Convention 2005).

C) The defendant is likely to be successful in its application because the claimant has the ability to comply with any order made for security for costs.

D) The defendant is likely to be successful in its application because it is not using the application to stifle the claimant’s claim.

E) The defendant is likely to be unsuccessful in its application because there is no reason to believe that the claimant will be unable to pay the defendant’s costs if ordered to do so.

A

A) The defendant is likely to be unsuccessful in its application because of its delay in making the application

(Correct. Although the defendant will likely be able to establish that one of the prescribed conditions for security for costs exists (CPR 25.13(2)(a)) and that the claimant has the ability (eg funds) to pay into court or to give a guarantee, the court is likely to refuse the application due to the application of the further discretionary factors . The defendant has left it until just two weeks before the trial to make the application when the facts justifying the order have been known from the outset ie that the claimant is out of the jurisdiction and in a location where enforceability of the costs order will be difficult, and that it has the funds to pay security. An application for security for costs should be made promptly and as soon as facts justifying the order are known and the court is likely to refuse to exercise its discretion to make the order on the basis that there has been a significant delay in making the application)

85
Q

A wealthy supermarket chain wants to obtain an interim injunction against a car dealer possessing the neighbouring land to prevent the car dealer from parking cars on what the supermarket considers to be its own land. The claim has a 60% chance of success. If the car dealer goes ahead with this parking, the supermarket will suffer losses of approximately £400 for each day of infringement, being the cost of the supermarket renting the required space from a different neighbouring property instead. The car dealer has substantial assets and available cash. Which of the following best explains why an application for an interim injunction is likely to fail?

A) The parking complained of has not taken place yet.

B) Damages will be an adequate remedy for the supermarket if the claim succeeds.

C) The supermarket will be unable to show the court that it will be able to honour the undertakings that are likely to be required of it.

D) The defendant has sufficient assets to pay the costs of opposing the application.

E) The claim does not have sufficient prospects of success.

A

B) Damages will be an adequate remedy for the supermarket if the claim succeeds

(The approach the court takes to determining whether to grant an interim injunction stems from the case of American Cyanamid Co v Ethicon Ltd [1975] AC 396 and in overview is: Step 1 - Is there a serious question to be tried? If not, an interim injunction is unlikely to be granted. But in this case, the claim has 60% prospects of success, so there is clearly a serious question to be tried, and it would be wrong to say that this claim does not have sufficient prospects of success. Step 2 - Would damages be an adequate remedy for a party injured by the court’s grant of, or failure to grant, an injunction? An injunction is unlikely to be granted unless damages would be inadequate for the applicant. In this case, there is nothing to suggest damages would be inadequate, and this is why an interim injunction is unlikely to be obtained. Examples of situations where damages would be inadequate are where the harm is irreparable or impossible to calculate – not the case here. If damages were to be inadequate for the applicant, you would consider the situation from the respondent’s point of view – is it possible to compensate the respondent if it turns out that an injunction should not have been granted (and could the applicant honour an undertaking to that effect)? This should be possible here from the facts we are given – the applicant appears to have sufficient money to honour an undertaking to pay the respondent damages and so it would be wrong to say that the supermarket will be unable to show the court that it will be able to honour the undertakings that are likely to be required of it. Step 3 - Where does the balance of convenience lie? Given the conclusions reached in relation to Step 2, this step is not very relevant. The other answers are wrong because interim prohibitory injunctions are ideal where the behaviour complained of has not yet taken place and because the respondent having sufficient assets to pay the costs of opposing the application does not that mean that it has good grounds to oppose it)

86
Q

When might the court order the parties to give disclosure?

A) At a case management conference or on a party’s application.

B) On a party’s application.

C) When giving directions on allocation, at a case management conference, or on a party’s application.

D) When giving directions on allocation.

E) At a case management conference.

A

C) When giving directions on allocation, at a case management conference, or on a party’s application.

87
Q

Kelly has brought a claim against Shazia for loss of profits following a breach of contract by Shazia. The claim is valued at £65,000. The court has allocated the claim to the multi-track and listed a CMC. When must Kelly file and serve her disclosure report?

A) At least 7 days before the CMC.

B) At least 14 days before the CMC.

C) At least 28 days before the CMC.

D) Kelly is not obliged to file or serve a disclosure report.

E) At least 21 days before the CMC

A

B) At least 14 days before the CMC

Correct. (CPR 31.5(3)).

88
Q

The judge in a multi-track case is likely to consider at the first CMC…

A) Whether or not to order standard disclosure.

B) Whether the parties have complied with their disclosure obligations.

C) What type of disclosure order to make.

D) Whether or not to order specific disclosure.

E) What type of disclosure order, if any, to make.

A

E) What type of disclosure order, if any, to make.

Correct. (CPR 31.5).

89
Q

Max has brought a claim against Bill following a road traffic accident. The damages for Max’s injuries are estimated at £35,000. The court has allocated the claim to the multi-track and listed a CMC. When must Max file and serve his disclosure report?

A) At least 14 days before the CMC.

B) At least 28 days before the CMC.

C) At least 7 days before the CMC.

D) Max is not obliged to file or serve a disclosure report.

E) At least 21 days before the CMC

A

D) Max is not obliged to file or serve a disclosure report

Correct. There is no obligation to file and serve a disclosure report in a personal injury claim.

90
Q

What is the usual order made in relation to disclosure in a claim allocated to the fast track?

A) Disclosure of unknown adverse documents.

B) No order for disclosure.

C) Standard disclosure.

D) There is no ‘usual order’.

E) The parties must file and serve documents they intend to rely on 14 days before the final hearing.

A

C) Standard disclosure

Correct – see the standard fast track directions (PD28).

91
Q

The approach recommended in this element for considering what a party should disclose under an order for standard disclosure is to consider…

A) Is it a document, does it fall within the test for standard disclosure (CPR 31.6)?

B) Is it a document, is it or was it in the party’s control, does the party wish to rely on it?

C) Is it a document, is it or was it in the party’s control, does it fall within a reasonable search (CPR 31.7)?

D) Is it a document, is it in the party’s control, does it fall within the test for standard disclosure (CPR 31.6)?

E) Is it a document, is it or was it in the party’s control, does it fall within the test for standard disclosure (CPR 31.6)?

A

E) Is it a document, is it or was it in the party’s control, does it fall within the test for standard disclosure (CPR 31.6)?

92
Q

Which of the following is not a document?

A) A text message.

B) An email.

C) A phone number memorised by one of the parties.

D) Information stored on a computer about when a document was created, and who by.

E) A voicemail.

A

C) A phone number memorised by one of the parties

Correct. See the definition of document (CPR 31.4). The other answers are all documents - information stored on a computer about when a document was created, and who by, is also included – this is called ‘meta data’.

93
Q

Which of the following is not prescribed as a relevant factor when deciding the reasonableness of a search for the purposes of standard disclosure (CPR 31.7)?

A) The nature and complexity of the proceedings.

B) The ease and expense of retrieval.

C) The significance of any document which likely to be located during the search.

D) The number of documents involved.

E) The resources of the party required to carry out the search.

A

E) The resources of the party required to carry out the search.

Correct. This is not stated in the rules as a relevant factor, but the court can still take it into account if it thinks it is appropriate to do so eg under the overriding objective. The list of factors in the rules (CPR 31.7) is not a list of all possible relevant factors.

94
Q

Which of the following falls outside of the concept of ‘control’ within the disclosure rules (CPR 31.8) as far as Party X is concerned?

A) A document obtained by Party X’s solicitor to be used as evidence in Party X’s case.

B) A document which was destroyed in a fire at Party X’s premises.

C) A document in the possession of a witness who is supporting Party X.

D) A document held by a director of Party X in his capacity as a director of Party X.

A

C) A document in the possession of a witness who is supporting Party X

Correct. It may be that this will be provided to Party X if Party X asks for it, and at that point it would become in Party X’s control. However, until that happens, it is not within the meaning of ‘control’ within the disclosure rules (CPR 31.8). The other documents are or were all in Party X’s control. Party X’s director would hold documents like the one described as agent for Party X. Party X would have a right to inspect / take possession of a document obtained by its own solicitor for use as evidence.

95
Q

The CPR lists 3 exceptions to the general right to inspect a document which has been disclosed. Which of the following IS NOT one of them?

A) It would be unreasonable to have to search for the document.

B) Allowing inspection would be disproportionate.

C) The disclosing party has a right / duty withhold inspection (eg it is privileged).

D) The document is no longer in the disclosing party’s control.

A

A) It would be unreasonable to have to search for the document

Correct. This is relevant to whether the document should be disclosed in the first place, but is not listed as an exception to the right to inspect (CPR 31.3).

96
Q

A company is in involved in litigation and has been obliged to disclose management accounts. Within the accounts are figures relating to directors’ salaries which are not relevant to the dispute and which are confidential. Which of the following is correct in relation to inspection and redaction of the accounts?

A) Inspection of the accounts need not be permitted because redaction is not practically possible.

B) Inspection of the accounts must be permitted. The commercially sensitive figures can be redacted because they are privileged.

C) Inspection of the accounts must be permitted. The confidential figures can be redacted because they are irrelevant and commercially sensitive.

D) Inspection of the accounts need not be permitted. The issue of redaction does not arise.

E) Inspection of the accounts must be permitted. The confidential figures can be redacted because they are commercially sensitive.

A

C) Inspection of the accounts must be permitted. The confidential figures can be redacted because they are irrelevant and commercially sensitive

Correct. You have correctly understood that redaction of commercially sensitive parts of a document is only permitted if those parts are irrelevant or privileged.

97
Q

The maxim ‘once privileged, always privileged’ encapsulates which legal principle in relation to privilege?

A) If a document is privileged, then that privilege cannot later cease / be lost as a result of actions of the disclosing party.

B) If privilege is established in relation to one type of document (for example, letters from a solicitor to a client) then it is established in relation to all documents in the same category (ie all letters from the solicitor to the client).

C) If a document is privileged in relation to one set of proceedings, it will be privileged in relation to later proceedings, even if those proceedings are entirely unconnected.

D) If a document is privileged in relation to one set of proceedings, it will be privileged in relation to later proceedings, if the two sets of proceedings concern substantially the same subject matter.

E) If a document is privileged in relation to disclosure, it will be privileged in relation to inspection.

A

C) If a document is privileged in relation to one set of proceedings, it will be privileged in relation to later proceedings, even if those proceedings are entirely unconnected.

98
Q

You are a litigation solicitor at Price Prior. You are instructed by Janet McCormack in relation to a claim she has against Dockland Removals Limited (“Docklands”). Docklands’ solicitor phones you a day before his client’s defence is to be served and asks for an extension of time of 14 days to file and serve its defence. You take instructions from your client who agrees to the extra time. You then phone up Docklands’ solicitor to let them know that the extension of time has been agreed. You make a note on one sheet of paper of the three conversations a) with Docklands’ solicitor, b) with your client, Janet McCormack and c) with Docklands’ solicitor again. The case comes to the disclosure stage and an order for standard disclosure is made, and you conclude that the sheet of paper must be disclosed. Is the sheet or any part of it privileged from inspection?

A) No part of the sheet of paper is privileged as its contents deal with the conduct of the case

B) The sheet of paper is not privileged as the dominant purpose of the creation of the piece of paper is to record a note of a telephone conversation with the other side’s solicitor which is not privileged.

C) The part of the sheet of paper concerning the conversation with your client is the only part of the sheet of paper which is privileged.

A

C) The part of the sheet of paper concerning the conversation with your client is the only part of the sheet of paper which is privileged.

Correct. The record of the conversation between solicitor and client will be subject to litigation privilege and/or legal advice privilege. The rest of the sheet of paper is not privileged. The dominant purpose test is not really relevant here: we are not looking at a single communication created for several purposes, which is when the ‘dominant purpose’ test would be useful. Here, we have distinct communications, each with its own clear purpose, and they can effectively be dealt with separately. In practical terms, this means letting the other side see the sheet of paper, but with the record of the conversation between solicitor and client ‘redacted’ ie hidden. Redaction is explained in the element ‘Inspection’.

99
Q

You are a solicitor at Price Prior and you act for Elizabeth Jones (‘EJ’). EJ has brought proceedings against Terry Val (‘TV’) for personal injury she sustained to her back as a result of a car crash she had with him in June of this year. You have drafted the Particulars of Claim and at paragraph 4 you state that Ms Jones has not sustained any previous injuries to her back prior to this accident. However, during your investigation of this matter, you come across a note from a doctor in Tanzania. The note shows that 12 months before the accident occurred, EJ sustained an injury to her back whilst climbing Mount Kilimanjaro. EJ had failed to report this to her British GP. The matter proceeds to disclosure and you include this note in the List of Documents and send the list for approval to your client (standard disclosure has been ordered). EJ telephones you the next day and says that under no circumstances should TV see this document. Which one of the following statements correctly describes the status of this document?

A) The doctor’s note need not be disclosed, so the question of inspection does not arise.

B) The doctor’s note must be disclosed and inspection permitted

C) The doctor’s note needs to be disclosed, but inspection can be refused because legal advice privilege applies.

D) The doctor’s note needs to be disclosed, but inspection can be refused because litigation privilege applies.

A

B) The doctor’s note must be disclosed and inspection permitted

Correct. The document is in the client’s control and is potentially adverse to the client, and therefore must be disclosed. No privilege applies - litigation was not reasonably in prospect at the time the note was written and neither does it meet the test for legal advice privilege (or any other privilege).

100
Q

You are a trainee in the litigation department at Price Prior. You attended and took notes at the case management conference in High Court proceedings issued against your client, Robinson Asset Management Limited (“RAM”) by Lawton Holdings Plc (“LH”). The case management conference was attended by you, your supervising partner, a representative from your client, RAM, counsel for your client, and also by a representative of LH, a partner and an associate solicitor from its representing solicitors (a firm called Taylor Dockett LLP), and counsel. The partner from Taylor Dockett solicitors has contacted you to say that unfortunately owing to an IT problem, they have lost the notes they made of the meeting and they have requested a copy of your notes. You believe that even if the court orders standard disclosure you are not obliged to provide Taylor Dockett with a copy of your notes because they are covered by litigation privilege. Is this true or false?

A) True

B) False

A

B) False

Correct. Pursuant to the case of Parry v Newsgroup Newspapers, a solicitor’s attendance note of a conversation between parties (ie between the solicitors for each party normally), or what happens at court, is not privileged since, notwithstanding that the note is a communication, there is no confidentiality in notes of matters at which both sides are present. Therefore it is likely that you will have to provide a copy of your notes as part of the disclosure and inspection stage of proceedings.

101
Q

Your opponent writes to you by a letter clearly marked ‘without prejudice save as to costs’. Can you show that document to the court?

A) It depends on the content of the letter

B) No.

C) Yes, but only on the question of costs.

D) Yes.

A

A) It depends on the content of the letter

Correct. If the letter is a genuine attempt at settlement, then it will be privileged, and can only be shown to the court when it comes to costs. If it is not a genuine attempt at settlement, then it is unlikely to be privileged at all. The fact that the document is labelled a particular way is not conclusive.

102
Q

An order for specific disclosure is an order that a party must….

A) Search for documents.

B) Search for documents and/or disclose specified documents.

C) Disclose specified documents.

D) Search for documents, disclose specified documents and/or allow inspection of specified documents.

A

B) Search for documents and/or disclose specified documents.

Correct. (CPR 31.12(2))

103
Q

Which of the following IS NOT required before the court is able to make an order for pre-action disclosure?

A) Disclosure of the documents is proportionate to the sums at stake in the proceedings.

B) The respondent is likely to be a party to subsequent proceedings

C) The applicant is also likely to be a party to those proceedings

D) If proceedings had started, the respondent’s duty by way of standard disclosure (CPR 31.6) would extend to the documents or classes of documents which the applicant seeks

E) Pre-action disclosure is desirable in order to (i) dispose fairly of the anticipated proceedings; (ii) assist the dispute to be resolved without proceedings; or (iii) save costs

A

A) Disclosure of the documents is proportionate to the sums at stake in the proceedings

Correct. This is not required before the court is able to make an order for pre-action disclosure. Although it is not a requirement, it would be a relevant consideration, although note also that the respondent can be compensated for costs of compliance by a suitable costs order (as explained in this element) and that proportionality is about more than just the sums in dispute (consider also the complexity of the dispute etc). All the other answers are required by the rules (CPR 31.16) before the court may make an order for pre-action disclosure. Note that even if all these factors are present, the court still has a discretion as to whether to order disclosure – it is not obliged to do so.

104
Q

An order for specific disclosure can be made…

A) At any time before proceedings have been issued.

B) At any time after proceedings have been issued

C) At any time after a disclosure order has been made and the time for compliance with that order has passed.

D) At any time before or after proceedings have been issued.

A

B) At any time after proceedings have been issued

Correct! Well done. An order for specific disclosure (CPR 31.12) can be made at any time after proceedings have been issued. Before proceedings, pre-action disclosure (CPR 31.16) would instead be considered.

105
Q

A Norwich Pharmacal order can only be sought against someone who:

A) has committed a wrong

B) has facilitated the wrongdoing

C) is more than a mere ‘witness’ or bystander

D) is associated with the wrongdoer

A

C) is more than a mere ‘witness’ or bystander

106
Q

In relation to both non-party disclosure (CPR 31.17), and also Norwich Pharmacal Orders, the likely costs order if the applicant succeeds in its application is…

A) That the applicant pays the respondent’s costs of the application.

B) That the respondent (the unsuccessful party) pays the applicant’s costs.

C) That the applicant pays the respondent’s costs of the application and of giving the disclosure / information.

D) That the respondent pays the applicant’s costs of the application, but the applicant pays the respondent’s costs of giving disclosure.

A

C) That the applicant pays the respondent’s costs of the application and of giving the disclosure / information.

107
Q

One of the prescribed conditions that must be satisfied before the court has a discretion to order non-party disclosure (CPR 31.17) is:

A) disclosure of the documents sought is necessary in order to dispose fairly of the claim or to save costs.

B) disclosure of the documents is necessary to avoid proceedings, encourage settlement or allow the proceedings to be conducted more efficiently.

C) disclosure of the documents sought is necessary in order to save costs.

D) disclosure of the documents sought is necessary because the documents cannot be obtained from a party.

A

A) disclosure of the documents sought is necessary in order to dispose fairly of the claim or to save costs

Correct – well done! Looking at the other answers: whether the documents can be obtained from a party (rather than non-party) is a relevant consideration, but does not form part of any of the conditions that must be satisfied.

108
Q

Which of the following statements best describes the costs rules of general application?

A) The court has limited discretion as to costs and will always adopt the starting position that the unsuccessful party pays the costs of the successful party only moving away from this if the conduct of the receiving party makes it fair and just to do so.

B) When deciding what costs order to make the court will have regard to three factors: the conduct of the party, the value of the claim and any settlement offers made.

C) The court has no discretion as to costs and is bound by the rule that the unsuccessful party will always pay for costs of the successful party.

D) The court has discretion as to costs but will adopt the starting position that the unsuccessful party pays the costs of the successful party only moving away from this if the conduct of the paying party makes it fair and just to do so.

E) The court has discretion as to costs with the general rule being that the unsuccessful party pays the costs of the successful party.

A

E) The court has discretion as to costs with the general rule being that the unsuccessful party pays the costs of the successful party.

This is correct. The court has discretion as to whether costs are payable, the amount of costs and when they are to be paid and the general rule on costs (CPR 44.2(2)) is correctly described in this answer. When making a costs order, the court will consider the conduct of both parties and not just the receiving party.

109
Q

The defendant to a large negligence claim has recently been served with the claim form and particulars of claim. There is no specific pre-action protocol governing the matter but to date the defendant has tried to avoid litigation by providing a detailed letter of response to the initial letter of claim, complying with all requests for information and disclosing key documents relevant to the dispute. It has also suggested some form of alternative dispute resolution (‘ADR’) on three separate occasions. All of these invitations to engage in ADR have been rejected outright by the claimant with no explanation given. How will the court view the claimant’s conduct when exercising its discretion as to costs?

A) If the defendant goes on to win at trial then the court is likely to exercise its discretion to deprive the claimant of some or all of its costs on the grounds that it has refused to agree to ADR.

B) If the claimant goes on to win at trial the court is likely to penalise the claimant for its refusal to engage in ADR by rejecting outright any claim for costs incurred during the pre-action stage.

C) The court will expect the claimant to actively engage in ADR but will not consider a refusal to do so to fall under any consideration of party conduct when exercising its discretion on costs.

D) If the claimant goes on to win at trial then the court is likely to exercise its discretion to deprive the claimant of some or all of its costs on the grounds that it has refused to agree to ADR.

E) The court will not expect the claimant to actively engage in ADR and so it is unlikely there will be any costs consequences of its refusal should it go on to win at trial.

A

D) If the claimant goes on to win at trial then the court is likely to exercise its discretion to deprive the claimant of some or all of its costs on the grounds that it has refused to agree to ADR

This is correct. When exercising its discretion on costs the court will have regard to a number of factors contained within the rules in relation to what order to make (CPR 44.2(4)) and in relation to the amount of costs to be awarded (CPR 44.4(3)). One factor common to both of these decisions is that the court will have regard to the parties’ conduct. An example of party conduct that might have an impact on costs is that of an unreasonable refusal by one party to engage in ADR (and this refusal does appear to be an unreasonable refusal).

110
Q

At the trial of a multi track contractual claim, the court makes its judgment in favour of the defendant. What is the most likely costs order that the court will make in relation to costs?

A) The claimant to pay the defendant’s costs on the standard basis.

B) The claimant to pay 100% of the defendant’s costs.

C) The claimant to pay 80% of the defendant’s costs.

D) The defendant to pay the claimant’s cost on the standard basis.

E) The claimant to pay the defendant’s costs on the indemnity basis.

A

A) The claimant to pay the defendant’s costs on the standard basis.

This is correct: the claim has been successful, and it is normal for the unsuccessful party to pay the costs of the successful party. This would be the ‘usual rule’ given that the application has been unsuccessful (CPR 44.2 and 44 PD 4.2). There is nothing on the limited facts to suggest any of the other costs orders, nor indemnity costs.

111
Q

Where costs are fixed by the CPR for a particular item, this means that…

A) …if the court awards costs for that item, the sum awarded will be as set out in the CPR, unless the court orders otherwise.

B) …if the court awards costs for that item, the sum awarded will be as set out in the CPR.

C) …a party is entitled to payment of those costs by the opponent, unless the court orders otherwise.

D) …if the court awards costs for that item, the sum awarded will be capped at the sum set out in the CPR, unless the court orders otherwise.

A

A) …if the court awards costs for that item, the sum awarded will be as set out in the CPR, unless the court orders otherwise

Correct. The fixed costs regime applies unless the court orders otherwise. The fixed costs regime is about deciding how much costs should be paid if they are going to be paid, not whether costs should be paid.

112
Q

If an application is due to be heard at 10am on Wednesday 5 November, when is the latest that a statement of costs (N260) can be filed and served in relation to that application?

A) 4pm on Tuesday 4 November

B) 10am on Monday 3 November

C) 4pm on Monday 3 November

D) 10am on Tuesday 4 November

A

D) 10am on Tuesday 4 November

Correct. The statement of costs must be filed and served party not less than 24 hours before the time fixed for the hearing.

113
Q

In which circumstances will costs be summarily assessed?

A) At the end of trials and interim hearings lasting not more than one day.

B) At the end of fast-track trials lasting not more than one day and at the end of interim hearings lasting not more than one day.

C) At the end of fast-track trials and at the end of interim hearings lasting not more than one day.

D) At the end of fast-track trials lasting not more than one day and at the end of interim hearings.

A

C) At the end of fast-track trials and at the end of interim hearings lasting not more than one day.

114
Q

A claimant makes a successful application to court to adduce evidence from a further witness of fact. The hearing of the application lasted for an hour and at the end of the hearing the judge ordered that costs be in the case. Which of the following statements best describes the effect of the costs order made by the judge?

A) The party in whose favour this order is made will recover the costs of the interim hearing from the other party regardless of who eventually wins at trial.

B) The decision about who pays the costs of the interim hearing of the defendant’s application is put off to a later occasion.

C) If the defendant is awarded costs at trial, the claimant does not have to pay the defendant’s costs of bringing its application.

D) The party who eventually gets its costs at trial (usually the successful party) will recover its costs of this interim hearing from the other party.

E) If the claimant is successful and receives an order that it should be entitled to its costs at the end of trial, it cannot include the costs of opposing the defendant’s application.

A

D) The party who eventually gets its costs at trial (usually the successful party) will recover its costs of this interim hearing from the other party

This is correct. The effect of an interim order of ‘costs in the case’ is that the party that gets its costs following the final hearing of the dispute will recover its costs of that interim hearing. Putting off costs until a later occasion (not necessarily final hearing) is known as ‘costs reserved’. Recovery of the costs following the hearing regardless of who wins at trial is known as ‘costs in any event’. A situation where a claimant is awarded costs at the end of trial but cannot include its costs of the interim hearing would be an order for the ‘defendant’s costs in the case’. A situation where a defendant is awarded costs at trial but the claimant does not have to pay the defendant’s costs of the interim hearing would be an order for the ‘claimant’s costs in the case’.

115
Q

A judge hearing a defendant’s application to extend the time for service of a defence has granted the defendant the full 28 day extension requested. In her judgment she is highly critical of the claimant for failing to agree to any extension and refers to aspects of the claimant’s conduct which have put the defendant in a difficult position in the proceedings so far. Given the above which of the following interim costs orders is the judge most likely to make.

A) The claimant to pay the defendant’s costs in any event

B) The defendant to pay the claimant’s costs in any event

C) The defendant’s costs thrown away

D) Costs in the case

E) No order as to costs

A

A) The claimant to pay the defendant’s costs in any event

This is correct. The judge has found in the defendant’s favour and is very critical of the claimant’s behaviour. The implication is that the judge has sympathy with the defendant and so is likely to award the costs it has incurred in bringing the application. The other options are not appropriate given the facts of the scenario. Costs thrown away relate to where a judgment or order is set aside, costs in the case would defer costs to final hearing and is less appropriate where the judge clearly favours one party’s position over the other and similarly no order as to costs ignores the fact that the judge is highly critical of the claimant’s conduct and sympathetic to the defendant’s position.

116
Q

A claimant has just succeeded in its interim application to amend its budget in light of significant developments in the proceedings. The hearing of the application lasted five hours and involved complex discussions on quantum and the value of forensic evidence. At the end of the hearing the judge ordered the claimant’s costs in any event. Which of the following statements best describes the effect of this order on the assessment of costs?

A) The judge will summarily assess the amount of the claimant’s costs immediately at the end of the hearing taking into consideration each party’s statement of costs and their respective brief submissions on what the costs order should be.

B) The judge will take the claimant’s statement of costs and unless it radically departs from the figures in the claimant’s budget the court will award those costs to be paid immediately or at the latest within 7 days of the order being made.

C) Given the complexities of the issues involved the judge is likely to involve an authorised court officer in the quantification of costs. Such assessment will take place immediately at the end of the hearing.

D) The judge will summarily assess the amount of the claimant’s costs immediately at the end of the hearing and then make a record of those costs so that they can be paid to the claimant following the final hearing of the dispute.

E) Given the complexities of the issues involved the judge is likely to involve an authorised court officer in the quantification of costs. Such assessment will take place at a later date when the officer will look at the costs incurred and decide how much should be paid.

A

A) The judge will summarily assess the amount of the claimant’s costs immediately at the end of the hearing taking into consideration each party’s statement of costs and their respective brief submissions on what the costs order should be.

This is correct. Following a ‘costs in any event’ order at a hearing which has lasted less than one day the court will go on to summarily assess the claimant’s costs before deciding on an amount. The defendant must then pay these costs within 14 days of the costs order being made (CPR 44.7). It is the judge that will summarily assess the costs and not an authorised court officer. When summarily assessing costs the court will be primarily concerned with each party’s statement of costs and their respective submissions. The influence of the budget at this interim hearing will be less important than when the judge is deciding costs of the whole proceedings following the final hearing of the dispute.

117
Q

Which of the following is not an example of the court’s power to manage cases? a) Extend time to comply with the court’s order to file a witness statement by 1st June. b) Require a party to attend court. c) Require a party to change its legal representative due to high legal costs. d) Require a party to file a costs budget. e) Require a party to take a step in the proceedings to further the overriding objective.

A) Require a party to file a costs budget.

B) Extend time to comply with the court’s order to file a witness statement by 1st June.

C) Require a party to change its legal representative due to high legal costs.

D) Require a party to attend court.

E) Require a party to take a step in the proceedings to further the overriding objective.

A

C) Require a party to change its legal representative due to high legal costs

Require a party to change its legal representative due to high legal costs is the correct answer as this is not an example of the court’s power to manage cases (CPR 3.1). The court will not interfere with the legal representative a party chooses. If the court considers that the legal representative is charging excessive legal costs, the court will address this when making orders as to the recovery of costs.
The other possibilities are all examples of the court’s power to manage cases: Extend time to comply with the court’s order to file a witness statement by 1st June. Require a party to attend court. Require a party to file a costs budget. Require a party to take a step in the proceedings to further the overriding objective.

118
Q

When can a court strike out a defendant’s statement of case? Choose the best answer.

A) The defendant has no real prospect of successfully defending the claim or issue and there is no other compelling reason why the case or issue should be disposed of at trial.

B) The defendant has failed to comply with a rule, practice direction or court order.

C) The defendant has not filed a defence.

D) The defendant has not responded to one out of five alleged breaches set out in the claimant’s particulars of claim.

E) The defendant has not filed a counterclaim.

A

B) The defendant has failed to comply with a rule, practice direction or court order

Correct answer: the court can strike out a defendant’s statement of case if the defendant has failed to comply with a rule, practice direction or court order. Failure to comply with a rule, practice direction or court order is one of the grounds for strike out (CPR 3.4(c)). The others are (a) that the statement of case discloses no reasonable grounds from bringing or defending the claim and (b) that the statement of case is an abuse of the court’s process or is otherwise likely to obstruct the just disposal of the proceedings.
The defendant has not filed a defence is incorrect. This is a ground for default judgment (rather than strike out) (CPR 12.3). It is easy to confuse strike out, summary judgment and default judgment. The defendant has no real prospect of successfully defending the claim or issue and there is no other compelling reason why the case or issue should be disposed of at trial is incorrect. These are the grounds for summary judgment (rather than strike out) (CPR 24.2). It is easy to confuse strike out, summary judgment and default judgment. The defendant has not responded to one out of five alleged breaches set out in the claimant’s particulars of claim is not the best answer. It is unlikely to amount to one of the grounds for strike out, for example, (a) where the statement of case discloses no reasonable ground for bringing or defending a claim. To fall within this ground it is more likely that a defendant would have only filed a bare denial with no other information. The defendant has not filed a counterclaim is incorrect; the defendant is not obliged to file a counterclaim in addition to a defence and, indeed, may not have a counterclaim against the claimant.

119
Q

Where a CPR / PD / court order requires a party to do something and stipulates the consequences of failing to comply, by how long can the parties extend the deadline?

A) 28 days (provided this does not put a hearing date at risk).

B) The parties cannot extend such a deadline.

C) There is no limit (provided the parties do not put a hearing date at risk).

D) 14 days (provided this does not put a hearing date at risk).

A

A) 28 days (provided this does not put a hearing date at risk).

120
Q

The claimant files its costs budget at court 60 minutes late. On the day of filing, the claimant’s solicitor had an accident on the way in to work and, although uninjured, had to attend hospital for several hours to get checked out. Once released from hospital, the solicitor filed the costs budget. The solicitor applies for relief from sanctions the following day. What factor is most likely to influence the court when considering whether to grant the relief from sanctions?

A) The breach prevents the case being conducted efficiently and at proportionate cost.

B) The breach is serious and significant.

C) The breach must be punished to enforce compliance with rules, practice directions and court orders.

D) The breach is not serious or significant.

E) The default occurred without good reason.

A

D) The breach is not serious or significant

The best answer is that the breach is not serious or significant. The case of Denton requires the court to look at three stages as part of its application of the provisions in relation to relief from sanctions (CPR 3.9). The first stage is to identify and assess the seriousness and significance of the failure to comply with the relevant rule, practice direction or court order. If the breach is neither serious nor significant, then relief should be granted. Here, the breach is not serious or significant and so relief from sanctions will usually be granted and it will not be necessary for the court to spend much time on the second and third stages.
The default occurred without good reason is not the best answer. The second stage of the Denton test is that if the breach is serious and significant, the court will consider why the default occurred. As stage one is satisfied, the court does not need to consider the second stage in any depth. In any event, here, there is good reason for the default.
The breach prevents the case being conducted efficiently and at proportionate cost is not the best answer. This is part of the third stage of the Denton test and deals with CPR 3.1(a). As stage one is satisfied, the court does not need to consider the third stage in any depth. In any event, here, CPR 3.1(a) does not point towards the refusal of relief as the breach did not prevent the litigation from being conducted efficiently and at proportionate cost, and did not imperil any further hearing date or otherwise disrupt the conduct of the litigation.
The breach must be punished to enforce compliance with rules, practice directions and court orders is not the best answer. As stage one is satisfied, the court does not need to consider the third stage in any depth. In any event, here, CPR 3.1(b) does not point towards the refusal of relief as the breach is near the bottom of the range of seriousness.
The breach is serious and significant is not the best answer. It is unlikely that the court would find this breach serious and significant.

121
Q

Putting aside personal injury claims / tenancy claims, the financial limit on the small claims track is:

A) £25,000

B) £10,000

C) £15,000

D) £50,000

E) £5,000

A

B) £10,000

Correct. Well done! This is the correct limit (CPR 26.6).

122
Q

Once Directions Questionnaires have been filed, the Court will always order that there be an allocation hearing. True or false?

A) True

B) False

A

B) False

Correct. The court will only order an allocation hearing if it considers it necessary (CPR 26.5(4)).

123
Q

When should the parties file and serve a Directions Questionnaire?

A) No later than the date specified on the Notice of Proposed Allocation.

B) At any point after service of the Defence but prior to the hearing of a Case Management Conference

C) At the same time as service of the Defence.

D) At least 3 days prior to the Case Management Conference.

A

A) No later than the date specified on the Notice of Proposed Allocation.

Correct. After service of the Defence, the court will provisionally decide the track most suitable for the claim and issue a notice of proposed allocation. This will require the parties to file and serve Directions Questionnaires and will include the date by which this must be done (CPR 26.3(6)).

124
Q

You are a solicitor at Price Prior and you act for Exclusive Builders Ltd (‘EBL’). EBL are claiming £27,500 in damages, interest and costs for breach of contract and negligence against Stanleys Steel Plc (‘SSP’) whom they employed to undertake the specialist steel work required in the construction of a large, state-of-the-art conference centre. The total amount of damages claimed includes £2,000 in interest and £1,580 in costs. SSP’s primary case is there has been no breach of contract or negligence; it has also pleaded that EBL is contributory negligent and the claim should be reduced by £15,000 as a result. EBL has appointed two experts; a surveyor, Mr Singh and quantum expert, Mr Bell. Similarly, SSP has appointed two experts Mr Smith, a surveyor and Mr Pagne, a quantum expert. Only Mr Bell and Mr Pagne will give oral evidence at trial. Trial is expected to last one day. What is the ‘normal track’ for this claim?

A) The normal track is the multi-track because 2 experts have been appointed by each party.

B) The normal track is the fast track.

C) The normal track is the multi-track because it is over £25,000.

D) The normal track is the multi-track because the value of the claim for allocation purposes is £42,500.

A

B) The normal track is the fast track.

Correct! Well done. The claim is for under £25,000. When assessing financial value for allocation purposes, the court will disregard any claim for interest, costs and any contributory negligence (CPR 26.8(2)). The maximum value for this claim for allocation purposes is therefore £23,920. The fast track is the normal track for claims of this value only where the trial is estimated to be one day or less and where oral expert evidence is limited to one expert per party in relation to any expert field and expert evidence is limited to two different expert fields (CPR 26.6(5)) – the trial length and evidence are so limited in this case.

125
Q

In relation to a claim allocated to the small claims track, the general rule is that even if successful at trial a party…

A) …cannot recover costs from an opponent.

B) …can only recover limited fixed costs, court fees, counsel’s charges and witness expenses from an opponent.

C) …can only recover limited fixed costs, court fees and witness expenses from an opponent.

D) …can only recover limited fixed costs from an opponent.

A

C) …can only recover limited fixed costs, court fees and witness expenses from an opponent.

126
Q

How are documents / statements from witnesses dealt with on the small claims track?

A) There is no order for disclosure or exchange of witness statements. Parties are required file and serve on every other party copies of documents they intend to rely upon no later than 14 days before the main hearing.

B) Standard disclosure is the ordinary order, to take place 28 days before trial. Witness statements are exchanged 14 days before trial in relation to any witness which is not attending the hearing. Witnesses attending the hearing do not need to provide statements in advance.

C) Standard disclosure is the ordinary order, to take place 28 days before trial. Witness statements are not used on the small claims track.

A

A) There is no order for disclosure or exchange of witness statements. Parties are required file and serve on every other party copies of documents they intend to rely upon no later than 14 days before the main hearing

Correct. This is the normal direction for a small claims track dispute.

127
Q

The parties must file agreed / proposed directions with the court….

A) At least 7 days before any CMC

B) At least 7 days before the first CMC

C) At least 14 days before the first CMC

D) At least 14 days before any CMC

A

A) At least 7 days before any CMC

Correct. The parties should attempt to agree them first. (CPR 29.4)

128
Q

The main purpose of a CMC is to…

A) Review the statements of case and ensure they are clear and complete.

B) Ensure the parties and their representatives are considering settlement.

C) Determine any issues which the parties wish to be determined.

D) Determine directions for the future conduct of the case.

A

D) Determine directions for the future conduct of the case.

Correct. Whilst the court will encourage settlement, and may assist with other issues, the court’s primary concern is directions (CPR 29.2(1)).

129
Q

When does the disclosure report need to be filed (assuming one is needed)?

A) In accordance with the directions made at the CMC

B) With the directions questionnaire

C) Not less than 7 days before the first CMC

D) Not less than 14 days before the first CMC

E) Not less than 21 days before the first CMC

A

D) Not less than 14 days before the first CMC

Correct. (CPR 31.5)

130
Q

Parties should only use page 1 of the Precedent H if:

A) The monetary value of the claim is less than £25,000 or if the party’s budgeted costs do not exceed £10,000.

B) The monetary value of the claim is less than £50,000 and if the party’s budgeted costs do not exceed £25,000.

C) The monetary value of the claim is less than £25,000 and if the party’s budgeted costs do not exceed £10,000.

D) The monetary value of the claim is less than £50,000 or if the party’s budgeted costs do not exceed £25,000.

A

D) The monetary value of the claim is less than £50,000 or if the party’s budgeted costs do not exceed £25,000.

131
Q

Budgets do not include:

A) Estimated opponent’s costs.

B) Solicitor’s charges.

C) Estimated future costs.

D) Disbursements.

E) Incurred costs.

A

A) Estimated opponent’s costs

Correct. Your opponent will set out their incurred and estimated costs in their own budget.

132
Q

In a claim valued at just over £100,000, the first CCMC has been listed for Friday 30 November. When does the costs budget need to be filed?

A) Friday 23 November

B) Friday 9 November

C) Thursday 8 November

D) Thursday 22 November

A

C) Thursday 8 November

Correct. Where the stated value of the claim is £50,000 or more, the budget must be filed not less than 21 days before the first CCMC. Counting the time correctly takes you to Thursday 8 November.

133
Q

In a claim valued at just over £100,000, the first CMC has been listed for Friday 30 November. By when does the budget discussion report need to be filed?

A) On Friday 23 November at the latest.

B) On Thursday 22 November at the latest.

C) On Friday 16 November at the latest.

D) On Thursday 15 November at the latest.

A

B) On Thursday 22 November at the latest

Correct. You have correctly understood when the budget discussion reports must be filed, and you have counted time correctly.

134
Q

Assume that budgets have been filed. Which of the following most accurately states when the court will make a costs management order (CMO)?

A) The court will make a CMO unless it is satisfied that the budgeted costs are in accordance with the overriding objective.

B) The court will make a CMO unless it is satisfied that the litigation can be conducted justly and at proportionate cost in accordance with the overriding objective without a CMO.

C) The court will only make a CMO if it concludes that the litigation will not be conducted justly and at proportionate cost in accordance with the overriding objective without a CMO.

D) The court will make a CMO in all cases (to which the costs management regime applies) in order to ensure that the litigation is conducted justly and at proportionate cost.

A

B) The court will make a CMO unless it is satisfied that the litigation can be conducted justly and at proportionate cost in accordance with the overriding objective without a CMO

Correct. This is what the relevant rule states (CPR 3.15(2)).

135
Q

Where a court has decided to make a Costs Management Order (CMO), how does it deal with a part of a party’s budget in relation to estimated costs which the other party has agreed (as recorded in the relevant Budget Discussion Report)?

A) The fact that a party has agreed part of the other party’s budget is not relevant to the exercise that the court carries out when making a CMO.

B) The CMO records that the relevant part of the budget has been agreed. The court will not change the agreed figure.

C) The CMO records that the relevant part of the budget has been agreed, unless the court wishes to substitute its own figure.

D) The court will take into account that the relevant part is agreed when deciding what would be an appropriate figure for that part CMO.

A

B) The CMO records that the relevant part of the budget has been agreed. The court will not change the agreed figure

Correct. The court cannot interfere with agreed parts of the budget: if it objects to the figure agreed, the court is likely to record a comment to that effect which a subsequent judge might take into account when it comes to assessment (CPR 3.15).

136
Q

If a judge is assessing costs on the standard basis at the conclusion of a matter, what impact does a Costs Management Order (CMO) have on her task?

A) The judge will cap the costs at the figures in the budget, unless there is good reason not to do so.

B) The judge will cap the costs at the figures in the budget.

C) The judge will assess the costs at the figures in the budget, unless the judge considers it unjust to do so.

D) The CMO is not relevant when it comes to assessing costs.

A

A) The judge will cap the costs at the figures in the budget, unless there is good reason not to do so

Correct. This is what the relevant rule states (CPR 3.18), when combined with the general principle that a party cannot recover a greater sum for costs from an opponent than it has in fact incurred.

137
Q

A party fails to file a budget by the deadline. What is the default consequence?

A) They will be treated as having filed a budget containing sums equal to those proposed by the other party.

B) They will be treated as having filed a budget providing nothing for the costs of the litigation.

C);The court will determine what figures should have been in the budget.

D) They will be treated as having filed a budget comprising only the applicable court fees.

A

D) They will be treated as having filed a budget comprising only the applicable court fees.

Correct! Well done (CPR 3.14).

138
Q

To which one of the following would the costs management regime likely apply, assuming each claim was commenced 2 months ago:

A) A personal injury claim in the County Court valued at £45,000.

B) A breach of contract claim in the High Court, seeking £25 million.

C) A debt claim in the County Court, when the debt is £24,000.

D) A negligence claim in the High Court, seeking sums to be assessed, but where the claim states that the value is £10 million or more.

A

A) A personal injury claim in the County Court valued at £45,000

Correct. This would appear to be a multi-track claim and, on these facts, does not fall into any of the exceptions to the costs management regime.

139
Q

A claimant pharmaceutical company claims damages in the approximate sum of £20,000 against its defendant former employee. The claim is based on the alleged infringement of intellectual property rights in relation to a drug developed by the defendant whilst under contract and in the employment of the claimant. For the claimant, there is an important point of principle at stake and, as such, despite the relatively low value of the claim, the claimant requires the oral evidence of three separate expert witnesses and estimates that the trial will last two days. Which of the following statements most accurately describes the reasons behind and likely allocation of this claim?

A) The claim is most likely to be allocated to the multi-track due to the importance of the claim to the claimant.

B) The claim is most likely to be allocated to the multi-track due to the complexity of the claim.

C) The claim is most likely to be allocated to the fast track due to the value of the claim.

D) The claim is most likely to be allocated to the fast track due to there being only two parties to the claim.

E) The claim is most likely to be allocated to the multi-track due to the estimated length of the trial and amount of expert evidence required.

A

E) The claim is most likely to be allocated to the multi-track due to the estimated length of the trial and amount of expert evidence required.

Correct. The fast track is the normal track for any claim which has a value of not more than £25,000 as here (CPR 26.6(4)(b)(i)), but only if the trial is likely to last no longer than one day and the oral expert evidence will be limited to one expert per party in each of no more than two fields (CPR 26.6(5)). As this is not the case in the claim here, the multi-track is the normal, and therefore likely, track for this claim (CPR 26.6(6)).

140
Q

A claimant’s breach of contract claim has been allocated to the multi-track. Seven days before the deadline for filing the budget (Precedent H) the claimant’s solicitors realise that they will not be in a position to file and exchange the costs budget in time because it has not been possible to work out all the costs for counsel and experts (as required by Precedent H). Leading Counsel on the case is currently involved in a complex trial overseas which is due to complete a few days after the deadline and as such has been uncontactable for a number of weeks. There have also been issues with the claimant’s expert witness who has been on holiday for the last 4 weeks. Therefore, the claimant’s solicitors feel it is necessary to make an application to the court for an extension of time of 14 days. Which one of the following statements is the most accurate as to the nature of the extension and whether it will be granted?

A) This is an application for relief from sanctions (CPR 3.9) and it is likely to be granted.

B) This is an in-time application for an extension of time (CPR 3.1(2)(a)) which is unlikely to be granted at all (even in part).

C) This is an in-time application for an extension of time (CPR 3.1(2)(a)) where although the court may grant the application, due to the fact that it will disrupt the conduct of the litigation, it is unlikely to grant it in full.

D) This is an in-time application for an extension of time (CPR 3.1(2)(a)) which is likely to be granted in full.

E) This is an application for relief from sanctions (CPR 3.9) and it is unlikely to be granted.

A

C) This is an in-time application for an extension of time (CPR 3.1(2)(a)) where although the court may grant the application, due to the fact that it will disrupt the conduct of the litigation, it is unlikely to grant it in full.

Correct. The application is an in-time application, the requested extension of time for filing the costs budget would take us to the same date as the deadline for filing the budget discussion report. In order to provide the parties with the opportunity meaningfully to discuss the costs budgets and file an agreed budget discussion report, (whilst not affecting the date for the CCMC and bearing in mind the Overriding Objective), the court is likely to grant the application but not provide the full extension of time sought by the applicant.

141
Q

At the trial of a multi-track case of professional negligence lasting for 3 days, the following costs order is made: ‘The Defendant to pay the Claimant’s costs of the proceedings on the standard basis’. What is the court likely to do in relation to ascertaining the precise sum of costs to be paid?

A) The court will decide the amount of costs there and then by conducting a detailed assessment of costs.

B) The court will order the costs to be the subject of detailed assessment, if not agreed.

C) The court will list a further hearing within 14 days to determine whether or not a detailed assessment of the costs will be required.

D) The court will make an order for fixed costs.

E) The court will make a summary assessment of the costs.

A

B) The court will order the costs to be the subject of detailed assessment, if not agreed

Correct answer: The court is most likely to order a detailed assessment of costs to take place in this multi-track case lasting for 3 days (CPR 44.6 and 44PD9.2). A summary assessment of costs is more likely to be ordered in a fast track case or at the conclusion of a hearing lasting not more than one day which does not apply here. A detailed assessment of costs by a costs officer takes place after the trial and costs are payable within 14 days of the date of the judgment if it states the amount (which it does not here) or, as in this case, if decided later, the date of the certificate which states the amount (CPR 44.7). Fixed costs (CPR 45) are not relevant to multi-track cases. The court is not likely to subject the costs to a detailed assessment or arrange a hearing of its own initiative.

142
Q

Your client is the defendant in a breach of contract claim. The defendant wants to make an application to the court for security for costs against the claimant. Service of the acknowledgment of service has just taken place, indicating that the defendant intends to defend the claim.

What is the most likely costs order which the court will make if the defendant’s application is unsuccessful on the basis of lack of evidence?

A) No order as to costs

B) A wasted costs order.

C) Costs of the application to the defendant, in any event.

D) Costs of the application to the claimant, in any event.

E) Defendant’s costs in the case.

A

D) Costs of the application to the claimant, in any event

Correct. This means the defendant will have to pay the costs of the unsuccessful application to the claimant. This would be the ‘usual rule’ given that the application has been unsuccessful (CPR 44.2 and 44 PD 4.2).

143
Q

The claimant in a multi-track claim relating to breach of contract is preparing for a costs and case management conference which takes place in a month’s time. The claim is for £850,000 and the parties are agreed on the scope of disclosure, the witness and expert evidence required, the deadlines for the service of this evidence and all other timetabling issues arising in the dispute. The one issue that is not agreed is the parties’ respective estimates of likely costs. What documents must be filed by the claimant prior to (and in relation to) the costs and case management conference?

A) The costs budget, the budget discussion report, the electronic disclosure questionnaire and the directions order.

B) The costs budget, the budget discussion report, the disclosure report and agreed directions.

C) The directions questionnaire, the costs budget, the budget discussion report, the disclosure report and agreed directions.

D) The directions questionnaire, the costs budget, the budget discussion report, the disclosure report and directions order.

E) The costs budget, the budget discussion report, the electronic disclosure questionnaire and proposed directions.

A

B) The costs budget, the budget discussion report, the disclosure report and agreed directions

CORRECT: This option is the most appropriate given that the parties appear to be in agreement over the areas to be covered by directions. The budget and budget discussion report (CPR 3.13), disclosure report (CPR 31.5(3)) and agreed directions (CPR 29.4) all need to be filed. The ‘directions questionnaire’ and ‘directions order’ are not documents prepared in anticipation of the costs and case management conference (26.3(6)).

144
Q

A claimant in a construction dispute is pursuing a claim for £450,000 against a defendant contractor. The claim arose due to losses suffered when a building project was not completed on time. The case has been allocated to the multi-track and the judge has made a direction for standard disclosure by list of documents. One of the documents in the claimant’s file is a draft letter written at the time of the building project by the managing director of the claimant to the defendant. The draft letter states that the completion date for the building project could be pushed back indefinitely to allow the defendant to carry out a proper job. This letter was never sent and the managing director of the claimant wants to know whether she can prevent the other side from seeing it. Will the defendant be entitled to inspect the letter?

A) No, because it is a document which supports the defendant’s case but it is covered by legal advice privilege.

B) No, because it is a draft document that was written before the dispute arose so it is not subject to standard disclosure. .

C) Yes, because it is an open document between the two parties and so it is not covered by legal professional privilege.

D) No, because it is a document which supports the defendant’s case but it is covered by litigation privilege.

E) Yes, because it is a document which supports the defendant’s case and is not covered by legal professional privilege.

A

E) Yes, because it is a document which supports the defendant’s case and is not covered by legal professional privilege.

CORRECT: The letter appears to be adverse to the claimant’s case (and so supports the defendant’s case) as it suggests there is flexibility over the completion date so undermining an element of the original claim. As such it is subject to standard disclosure and does not fall under either litigation privilege or legal advice privilege. This letter is open to inspection by the defendant.

145
Q

A claimant is preparing for trial when the existence of a document that should have been disclosed to the defendant comes to light for the first time. At the time of disclosure, the claimant had carried out a reasonable search and properly carried out its disclosure obligations, but this document had not been revealed and therefore had not been disclosed.

What should the claimant do?

A) Tell the court about the document.

B) Disclose the document to the defendant.

C) Apply to the court for permission to disclose the document (late).

D) Present the case to the court at trial using the document but making clear that it has not previously been disclosed.

E) Present the case to the court at trial without referring to the document.

A

B) Disclose the document to the defendant

CORRECT: The duty of disclosure continues during proceedings and the duty is to disclose the document to every other party (CPR 31.11). Simply not referring to the document at trial is not an option – it could be that the document is one that is helpful to the other parties, so this would be unfair. There is no need to apply to court to disclose the document. However, if the claimant wants to rely on the document, the claimant may need to apply to court for permission to do so (CPR 31.21) as a party may not rely on any document which he fails to disclose or in respect of which he fails to permit inspection unless the court gives permission.

146
Q

A claimant is pursuing an action for negligence against their solicitors in the High Court. The claim has been allocated to the multi-track and the court has ordered standard disclosure. One document being considered for disclosure is an email from the managing director at the claimant to the other members of the board of directors. The first 5 paragraphs report that month’s profit figures for the claimant’s business - the managing director has sent such monthly reports for many years. The claimant’s profitability is an issue in dispute in the proceedings. The remainder of the email reports back from a recent meeting with solicitors and sets out the contents of the advice that was given by the solicitors to the claimant at that meeting. Which of the statements below is the most accurate as to the claimant’s disclosure obligations?

A) The claimant will not be required to disclose the document as it does not meet the test for standard disclosure.

B) The claimant will be required to disclose the document but can withhold inspection of the document on the grounds of legal professional privilege.

C) The claimant will be required to disclose the document and make it available for the defendant to inspect unredacted.

D) The claimant will be required to disclose the document and it cannot be withheld from inspection or redacted because the claimant has waived privilege over the document.

E) The claimant will be required to disclose the document and it cannot be withheld from inspection but parts that repeat legal advice will likely be able to be redacted as being privileged.

A

E) The claimant will be required to disclose the document and it cannot be withheld from inspection but parts that repeat legal advice will likely be able to be redacted as being privileged.

CORRECT: This document satisfies the test for standard disclosure. The document as a whole will be available for inspection. However, the parts that repeat the legal advice will be redacted as it will be subject to legal advice privilege – it is a confidential communication between a lawyer and a client where the client is repeating legal advice given to them to the rest of the directors on the board. Following the principles in the “Good Luck” case, the client effectively acts as the lawyer’s agent when he repeats the advice to the rest of the board and therefore this falls within the definition of legal advice privilege.

147
Q

A claimant driver is bringing proceedings against a defendant driver following a road traffic accident. The defendant has admitted liability and the claim is proceeding in relation to quantum only. The accident was captured on video by another driver who provided a copy of the video to the claimant, but the claimant deleted this by error before the proceedings started and does not have the contact details of the party that originally captured it. The court has ordered standard disclosure.

Which of the following best explains why the claimant is not obliged to disclose this video to the defendant?

A) The video is no longer in the claimant’s control.

B) The disclosure obligation does not extend to videos.

C) The video was created by a third party.

D) The identity of the ‘author’ of the video (the person that captured it) is unknown.

E) It is not something the claimant wishes to rely on, nor is it supportive of the defendant’s case or adverse to either party’s case.

A

E) It is not something the claimant wishes to rely on, nor is it supportive of the defendant’s case or adverse to either party’s case.

Correct. The obligation to give standard disclosure extends to videos – these are ‘documents’ (CPR 31.4). If a document is or was in the claimant’s control, it does not matter that it was created by a third party or that the author is unknown. The disclosure obligation extends to documents which are no longer in the claimant’s control. However, in this case liability has been admitted therefore it does not appear that this document is something which the claimant wishes to rely on, nor is it supportive of the defendant’s case or adverse to either party’s case which is now proceeding only in relation to quantum.

148
Q

A witness statement provided by the claimant includes a reference to a letter setting out Counsel’s advice to the claimant. Will the defendant be able to inspect the letter?

A) No because it is a draft document that was written before the dispute arose so it is not subject to standard disclosure.

B) Yes, as a party may inspect a document that is referred to in a witness statement.

C) No, as only the claimant has the right to decide whether or not to waive privilege over this document.

D) No, as Counsel’s advice is covered by litigation privilege.

E) Possibly, depending on the manner in which, and reasons for which, the advice is referred to in the witness statement.

A

E) Possibly, depending on the manner in which, and reasons for which, the advice is referred to in the witness statement.

Correct. A party can inspect a document referred to in a witness statement (CPR 31.14). This is subject to the usual rules on privilege (which prevent a right to inspect), but privilege may have been waived depending on the manner in which, and reasons for which, the advice is referred to in the witness statement.

149
Q

When does exchange of witness statements usually take place?

Immediately after the defendant has filed its defence.

Immediately after the parties have filed directions questionnaires.

Immediately after disclosure and inspection.

Immediately after exchange of expert evidence.

Immediately after allocation and case management.

A

Immediately after disclosure and inspection.

The court will usually give directions for the exchange of witness statements at the allocation and case management stage and therefore exchange will not take place beforehand e.g. after the defendant has filed its defence or the parties have filed directions questionnaires. Exchange of witness statements is generally the step in the action that follows disclosure and inspection. This is because it is useful to review all disclosure documents prior to finalising witness statements as the witnesses may need to refer to the documents in their evidence.

150
Q

Which of the following does the court have no power to control (in relation to witness statements)?

None of the other answers is correct – the court has the power to control all of those.

The number of witness statements.

The issues to be covered in witness statements.

The length of witness statements.

A

None of the other answers is correct – the court has the power to control all of those

Correct. The court can control the issues to be covered in witness statements, together with their number and length.

151
Q

The claimant calls a witness to give evidence at trial. How will the witness statement be used by the claimant at trial?

As written only evidence

For cross-examination

As evidence in chief

For re-examination by the defendant

For oral examination by the claimant

A

As evidence in chief

Any fact which needs to be proved by evidence of witnesses at trial will be by oral evidence (CPR 32.2(1)(a)). However, a witness statement usually stands as evidence in chief at the trial (CPR 32.5(2)) so that in this scenario the claimant would not have to take oral evidence from the witness. The witness is then cross-examined by the other party, here the defendant, and re-examined by the claimant.

152
Q

In what way is witness evidence for an interim hearing different to that for a trial?

A witness statement for an interim hearing does not necessarily need to be by an individual – it can be by a company.

A witness statement for an interim hearing requires different wording for the statement of truth.

A witness statement for an interim hearing has extra paragraphs relating to the reason for the statement and the position taken in relation to the application.

The written evidence for an interim hearing should be contained within an application notice or a statement of case.

A

A witness statement for an interim hearing has extra paragraphs relating to the reason for the statement and the position taken in relation to the application

This is correct. In relation to the other answers: - There is no difference in the statement of truth - The evidence can be within the application notice if it is short or it may already be contained in a statement of case, but it can also be in a witness statement instead. - A witness statement is always given by a specific individual, even if it is being given on behalf of a party which is a company or partnership.

153
Q

Near the start of a witness statement, the witness will usually include a paragraph stating:

I make this witness statement from matters within my knowledge or belief save where the contrary appears. Where I refer to matters of which I have been told by others, those matters are true to the best of my knowledge and the source of the information appears.

This witness statement is a true statement of the facts as I perceived them. Nothing in this witness statement is based on information provided to me by others.

I believe that the facts stated in this witness statement are true. I believe that the facts stated occurred as stated, and where there is a range of opinions on matters described, I state the range of opinions to the best of my ability.

I believe that the facts stated in this witness statement are true. Where I have consulted others in relation to this witness statement, there are views are also stated to the best of my belief.

A

make this witness statement from matters within my knowledge or belief save where the contrary appears. Where I refer to matters of which I have been told by others, those matters are true to the best of my knowledge and the source of the information appears.

154
Q

What is the correct form of statement of truth for a witness statement given by the managing director of the Claimant in support of the Claimant’s claim?

I believe that the facts stated in this statement of case are true. I understand that proceedings for contempt of court may be brought against anyone who makes, or causes to be made, a false statement in a document verified by a statement of truth without an honest belief in its truth.

I believe that the facts stated in this witness statement are true. I understand that proceedings for contempt of court may be brought against anyone who makes, or causes to be made, a false statement in a document verified by a statement of truth without an honest belief in its truth.

I believe that the facts stated in this witness statement are true.

The Claimant believes that the facts stated in this witness statement are true. I understand that proceedings for contempt of court may be brought against anyone who makes, or causes to be made, a false statement in a document verified by a statement of truth without an honest belief in its truth.

A

I believe that the facts stated in this witness statement are true. I understand that proceedings for contempt of court may be brought against anyone who makes, or causes to be made, a false statement in a document verified by a statement of truth without an honest belief in its truth.

155
Q

Which type of application needs evidence by way of affidavit rather than witness statement?

An application for an interim prohibitory injunction.

An application for a search order.

An application to vary a direction.

An application for summary judgment.

A

An application for a search order

Correct. Well done! The other applications listed do not require evidence in the form of an affidavit.

156
Q

Which type of application needs evidence by way of affidavit rather than witness statement?

An application for relief from sanctions.

An application for an interim payment.

An application to strike out a statement of case.

An application for a freezing order.

A

An application for a freezing order.

Correct. Well done! The other applications listed do not require evidence in the form of an affidavit.

157
Q

An important difference between an affidavit and a witness statement is that…

Proceedings for contempt of court can be based on any error in an affidavit, whereas in the case of a witness statement, it is only contempt of court if a witness does not have an honest belief in what is stated.

An affidavit is directly transcribed from an oral account, but a witness statement is not always so transcribed.

The form of statement of truth on an affidavit is different.

An affidavit must be sworn before a person authorised to administer affidavits
Correct
Correct. Well done! In terms of the other answers: - there is no statement of truth in an affidavit. - what is said in relation to proceedings for contempt of court is roughly correct in relation to witness statements, but incorrect in relation to affidavits. - it is wrong to say that an affidavit is directly transcribed from an oral account.

A

An affidavit must be sworn before a person authorised to administer affidavits

Correct. Well done! In terms of the other answers: - there is no statement of truth in an affidavit. - what is said in relation to proceedings for contempt of court is roughly correct in relation to witness statements, but incorrect in relation to affidavits. - it is wrong to say that an affidavit is directly transcribed from an oral account.

158
Q

All evidence that is relevant to the facts is admissible in civil proceedings. However, there are special procedural safeguards built into the rules and in the Civil Evidence Act 1995 relating to the admissibility of which type of evidence?

Opinion evidence

Oral evidence

Expert evidence

Hearsay evidence

Evidence which is privileged

A

Hearsay evidence

Hearsay evidence is the correct answer. Hearsay evidence is considered less reliable than direct evidence and so there are procedural safeguards to ensure hearsay can be carefully scrutinised eg by requesting particulars of the hearsay or by challenging its weight (Civil Evidence Act 1995 and CPR 33).
Opinion evidence is not the correct answer; the special rules relating to opinion evidence are contained in the Civil Evidence Act 1972. The element on expert evidence contains more details.
Evidence which is privileged is not the correct answer; the special rules for privilege are contained in case law. The elements on disclosure and inspection contain more details.
Oral evidence is not the correct answer; there are not special ‘evidential’ rules, but procedural requirements regarding oral evidence of witnesses of fact (CPR 32) and requirements regarding oral evidence of experts (CPR 35).
Expert evidence is not the correct answer; there are not special ‘evidential’ rules, but procedural requirements regarding expert evidence (CPR 35).

159
Q

How does a party give notice to the other party of hearsay that appears in the witness statement of a witness giving oral evidence at trial?

Exchange witness statements and provide formal notice of the hearsay.

No notice is required.

Exchange witness statements

Provide formal notice.

Telephone the other party.

A

Exchange witness statements

The correct answer is exchange of witness statements. No ‘formal’ notice is required as notice is deemed on exchange. This means it is incorrect to say that it is necessary to exchange witness statements and provide formal notice of the hearsay, or to telephone the other party, or that no notice is required, or that formal notice is required

160
Q

A man is giving oral evidence at trial on behalf of the claimant. He states to the court: “during the meeting, the defendant and I reached an agreement that the price would be £100 per item”.

Is this hearsay evidence?

It is hearsay because it is an oral statement.

It is not hearsay. He is making a statement in court of facts that someone else witnessed.

It is hearsay. It is being adduced in court to prove the truth of the matter stated.

It is not hearsay. He is making a statement in court of facts that he witnessed himself.

It is hearsay. He is repeating a statement that someone else made out of court.

A

It is not hearsay. He is making a statement in court of facts that he witnessed himself.

This is the correct answer. John’s statement is not hearsay. He is making a statement in court of facts that he witnessed himself.

161
Q

In a road traffic accident a driver suffers several injuries including brain damage, a punctured lung and a broken leg. What expert evidence is admissible in the personal injury case at court?

The orthopaedic surgeon giving an opinion on the implications of the driver’s punctured lung.

The orthopaedic surgeon conveying perceived facts.

The orthopaedic surgeon giving an opinion on the impact of the brain damage on the driver.

The orthopaedic surgeon giving an opinion on the relevant law.

The orthopaedic surgeon giving an opinion on the driver’s broken leg.

A

The orthopaedic surgeon giving an opinion on the driver’s broken leg.

This is the correct answer. The general rule is that opinions are not admissible. An exception is expert evidence. An expert must give an opinion on a matter upon which the expert is qualified. An orthopaedic surgeon deals with the skeleton and joints and so is an expert in giving an opinion on the driver’s broken leg. An orthopaedic surgeon is not an expert in a punctured lung or brain damage; these injuries would be outside the orthopaedic surgeon’s expertise. The judge is the expert in law. Another exception to the general rule is perceived facts. The orthopaedic surgeon is not a witness conveying relevant facts personally perceived by the expert.

162
Q

The defendants solicitor asks the court for permission to use an actuarial expert to assist with calculating damages. Which of the following is correct?

The court’s permission is needed to instruct the actuarial expert.

The court’s permission to call an expert is usually sought by the parties at the pre-action stage.

The actuarial should try to maximise the defendant’s claim for damages.

If the defendant’s solicitors do not exchange the expert’s report by the date set out in the court’s directions, the evidence cannot be used without the court’s permission.

The defendant’s solicitors must provide the name of the actuarial expert in the directions questionnaire.

A

If the defendant’s solicitors do not exchange the expert’s report by the date set out in the court’s directions, the evidence cannot be used without the court’s permission

This is the correct answer. An expert’s report must be exchanged by the date set out in the court’s directions otherwise the evidence cannot be used without the court’s permission (CPR 35.13). Permission (CPR 35.4) is to call an expert or to rely on the written expert’s report at trial; permission is not needed to ‘instruct’ the expert. Permission is usually sought in the directions questionnaire and will be dealt with in the directions issued at the case management stage. The name of the actuarial expert does not need to be provided in the directions questionnaire unless it is practicable to do so. The duty of the expert will be to the court and this overrides any obligation to the defendant’s solicitor to maximise the claim.

163
Q

A single joint expert has been instructed following a direction being made by the court in relation to this expert evidence. What should the single joint expert have received in addition to the instructions to provide a report?

An order confirming the date that the report of the expert shall be filed at court sent by the court.

An order confirming the date that the report of the expert shall be filed at court from the claimant.

An order to prepare a statement of the issues on which the experts agree and which they disagree.

An order confirming the date that the report of the expert shall be filed at court from the defendant.

An order confirming a direction for a discussion of experts.

A

An order confirming the date that the report of the expert shall be filed at court from the claimant

This is the correct answer. Where there is an order requiring an act by an expert, the party instructing the expert must serve the order on the expert (35 PD 8). In the case of single joint experts, this is the claimant (not the defendant or the court). A discussion of experts and a statement of issues would not be appropriate for a single joint expert, but are tools used to reduce the issues where each party instructs their own expert.

164
Q

The claimants solicitor has served extensive questions on the defendant’s expert within 28 days of receiving the expert’s report. The defendant’s expert believes that it would take a disproportionate amount of time to answer the questions and that the questions are simply cross-examination. What should the defendant’s expert do next to address its concerns?

The defendant’s expert should only answer a proportionate number of the questions.

The defendant’s expert should answer the questions as soon as possible.

The defendant’s expert should ignore the request to answer questions.

The defendant’s expert should file a written request with the court for directions.

The defendant’s expert should write to the claimant’s solicitor refusing to answer the questions.

A

The defendant’s expert should file a written request with the court for directions

This is the best answer. The rules allow an expert to submit written requests for directions to the court to help them carry out their task (CPR 35.14). This would be better than refusing to answer the questions, answering a number of questions or ignoring the request because the rules do allow the claimant to ask the expert questions within 28 days of the expert’s report (CPR 35.6). The expert’s answers become part of the report and so if the expert objects to the questions it is important that the issue is dealt with by the court rather than simply answering the questions.

165
Q

The parties in a breach of contract claim have both just received the final report from the single joint expert instructed to advise on the extent of losses suffered by the claimant as a result of the defendant’s breach. The claimant is unhappy with the expert’s conclusions and considers the approach taken to calculating those losses to be unclear and potentially incorrect.

Which of the following best describes what the claimant should do next to challenge the expert and the content of the report?

The claimant should wait until the final hearing of the dispute when it will be able to cross-examine the expert and challenge the approach taken to the calculation of loss.

The claimant should put written questions to the expert asking for clarification about the approach taken to the calculation of loss.

The claimant should immediately instruct its own expert to advise on loss to allow for enough time for that evidence to be adduced at final hearing.

The claimant should make an application that the instructions to the expert are disclosed so that it can ensure that they contain nothing inaccurate and are not incomplete.

The claimant should make an application to court asking that the expert’s report be struck out with immediate effect.

A

The claimant should put written questions to the expert asking for clarification about the approach taken to the calculation of loss

CORRECT: On receiving the report from the single joint expert the claimant should put written questions to the expert (CPR 35.6). This will help it understand how the expert came to their conclusions. It is not appropriate for the report to be struck out as the court’s power relates to striking out a statement of case and not an expert’s report (CPR 3.4). The claimant may indeed want to cross-examine the expert at the final hearing but does not need to wait until then before taking pro-active steps. The claimant cannot rely on the evidence of a new expert without the court’s permission and it is premature to seek this (CPR 35.4). The claimant will have seen the instructions to the single joint expert (CPR 35.8).

166
Q

A claim has been issued for non-payment of goods delivered by the claimant to the defendant on 8th February. The parties’ written contract stipulates delivery on 1st February. The claimant’s Finance Director made an oral variation to the contract for delivery to be on 8th February. The draft witness statement of the Managing Director (not the Finance Director) includes the paragraph: ‘The Finance Director told me that she spoke to the defendant to change the date of delivery of the goods from 1st February to 8th February and the defendant agreed to the change’.

Which of the following statements best describes how the claimant should deal with / improve upon this evidence?

Retain the paragraph from the witness statement and obtain a witness statement from the Finance Director.

Retain the paragraph in the witness statement and serve a notice that the claimant intends to rely on the evidence at trial.

Remove the paragraph from the witness statement.

Retain the paragraph in the witness statement.

Remove the paragraph from the witness statement, place it in a hearsay notice and serve the hearsay notice on the defendant.

A

Retain the paragraph from the witness statement and obtain a witness statement from the Finance Director.

CORRECT: The paragraph in the witness statement contains hearsay evidence which is an oral or written statement, made out of court, adduced in court to prove the truth of the matter stated. (Hearsay evidence can sometimes be understood as ‘back-up’ evidence. In other words, the witness has a view about the case and someone ‘says’ something, repeated in the witness statement which ‘backs up’ the witness’s point of view). Hearsay evidence is admissible in civil proceedings by s.1 Civil Evidence Act 1995 (CEA 1995) and so there is no need to remove it. However, if a witness statement can be obtained from the Finance Director, then this is much more persuasive and valuable. It makes the inclusion of the paragraph in the witness statement of the Managing Director much less necessary, but there may still be some benefit in retaining it.
Note that even if it was retained, there would be no need to serve a separate notice that the claimant intends to rely on the evidence at trial.

167
Q

In a breach of contract claim, the parties are due to exchange witness statements by 27 March. On 20 March, the Claimant receives notification that, due to the unavailability of a witness, it will not be possible to contact the witness and finalise her statement until 2 April. The next direction after exchange of witness statements is for experts to report, in late June. Which of the following statements best describes what Claimant should do?

Ask the Defendant for an extension of time for exchange of witness statements.

Notify the other side that the witness statements will be ready for exchange on 2 April and inform them that the date for exchange of witness statements will need to be delayed until then.

Notify the court that the witness statements will be ready for exchange on 2 April and inform them that the date for exchange of witness statements will need to be delayed until then.

Make an immediate application to extend the time for service of the witness statements until 2 April.

Serve the witness statements in their current form.

A

Ask the Defendant for an extension of time for exchange of witness statements

Correct. It is important not to miss the deadline because if a witness statement for use at trial is not served in respect of an intended witness within the time specified by the court, then the witness may not be called to give oral evidence unless the court gives permission (CPR 32.10) – ‘notifying’ the court or the opponent is not adequate to avoid the sanction. An application to the court should therefore be made before any deadline expires. However, the other side has the power to grant the extension sought, and this would avoid the need for a court application, so this should be explored first. Witness statements should not be served in draft form, as the draft could give away something prejudicial.

168
Q

A witness of fact is called to give evidence in the trial of a claim brought by a claimant property development company against a defendant accountancy firm for losses arising out of alleged negligent advice that was given by the defendant. The witness was the Financial Director of the claimant company at the time the alleged negligent advice was given. The witness’ statement includes the following sentence: ‘At the time the advice was given, the financial assessment provided by the defendant did not accurately reflect the claimant’s financial position’. Is this evidence likely to be admissible?

The sentence is likely to be inadmissible opinion evidence.

The sentence is likely to be admissible evidence of fact.

The sentence is likely to be admissible ‘perceived fact’ evidence.

The sentence is likely to be admissible expert opinion evidence.

The sentence is likely to be inadmissible privileged evidence.

A

The sentence is likely to be admissible evidence of fact

CORRECT: The sentence is evidence of fact given by the witness who would have been involved in the events at the time and had the knowledge (as Financial Director) to state that the financial assessment was not accurate. The witness would no doubt go on to give detail of where and why elements of the advice given were inaccurate, which again would be matters of fact. As such, the sentence does not consist of an opinion whether admissible or inadmissible ie it is not based on the witness’s value judgement of the facts. The sentence does not contain any information which falls under any head of privilege (privileged information is, in any event, admissible, if the party/ies entitled to claim privilege wish to waive this).

169
Q

The managing director of a software company is providing evidence in support of an interim application for a search order. In what manner will that evidence need to be ‘signed off’?*

With a statement of truth by the managing director on behalf of the company.

With a statement of truth on the part of the managing director personally.

With a jurat by the managing director on behalf of the company, sworn before someone who is authorised to administer affidavits.

With a jurat on the part of the managing director personally, sworn before someone who is authorised to administer affidavits.

With a statement of truth by the company.

A

With a jurat on the part of the managing director personally, sworn before someone who is authorised to administer affidavits.

Correct, well done! An affidavit is needed because the application is for a search order. An affidavit is signed off with a jurat rather than a statement of truth, and must be sworn before someone who is authorised to administer affidavits. An affidavit needs to be signed by someone personally – it is not given by a company or partnership (this is also true of witness statements).

170
Q

Under Part 36, in the absence of agreement in writing, how long does a defendant have to pay the offered sum of money to claimant if it accepts the offer?

14 days

7 days

28 days

21 days

Unless the offer states the period, there has been no effective offer.

A

14 days

Correct. (CPR 36.14).

171
Q

Manchester Hospitals NHS Trust has recently instructed you in respect of proceedings, issued against it by Herbert, for damages for clinical negligence. On Friday last week, Manchester Hospitals NHS Trust served its defence within the correct time limits. The claim is likely to be allocated to the multi-track. Manchester Hospitals NHS Trust wants to make a Part 36 offer but wants it to be a time limited offer ie only open for acceptance for a specified period of time. It asks you for advice on whether it is possible to make a time limited Part 36 offer.

What ONE of the following is the BEST advice in these specific circumstances?

Yes it is possible to make a time limited offer. The offer can be automatically withdrawn in accordance with its terms. There can be a term which time limits the Part 36 offer. This time limit can be of any period including less than the relevant period.

No it is not possible to make a time limited offer. Once a Part 36 offer is made it cannot be withdrawn under any circumstances.

No it is not possible to make a time limited offer. You would have to withdraw the offer by serving notice of withdrawal.

Yes it is possible to make a time limited offer. The offer can be automatically withdrawn in accordance with its terms, at the end of a specified period of time. However, the time limit must be after the expiry of the relevant period.

A

Yes it is possible to make a time limited offer. The offer can be automatically withdrawn in accordance with its terms, at the end of a specified period of time. However, the time limit must be after the expiry of the relevant period.

Correct. (CPR 36.9(4)(b)). Practically, this means that the Part 36 offer will state that it will be withdrawn by a set date or time, after the expiry of the relevant period, if it has not been accepted.

172
Q

Clinton Hart is bringing a claim for personal injury against Simon Malik. The claim arises out of a road traffic accident.

The case is issued in the High Court. Simon has admitted liability; the only matter in issue is the amount of damages. The case has been provisionally allocated to the multi-track and Directions Questionnaires are due to be filed next week. Clinton alleges that he has suffered permanent injuries, he cannot work, has mobility issues and needs a significant amount of care. Clinton’s medical evidence supports this and on full valuation the claim would be worth in the region of £1 million. Simon has made a Part 36 offer of £750,000. The relevant period is 21 days. This offer reflects Simon’s solicitor’s valuation of the claim as they feel that the care element of the claim is overvalued. The Part 36 offer was made 10 days ago and has not yet been accepted. Today, instructing solicitors have received an e-mail with an attachment. The attachment is a recording of Clinton playing football for his local football team during which he scores a goal with an overhead kick. There is no evidence of any injury. The sender is willing to provide a witness statement in support.

Advise the solicitors for Simon on the BEST course of action to adopt in relation to his Part 36 offer in light of the new evidence from the following options:

All Simon needs to do is serve a notice of withdrawal and it will take effect immediately.

Simon should do nothing.

As the relevant period has not expired, Simon’s Part 36 offer can only be withdrawn with the permission of the court. Therefore, Simon should apply immediately to the court for permission to withdraw the offer.

Simon should immediately serve a notice of withdrawal. It would then be for Clinton to decide whether or not to serve notice of acceptance. If Clinton did serve notice of acceptance Simon should apply to the court for permission to withdraw the offer within 7 days of Clinton’s notice of acceptance. Simon would argue that the court should grant permission as there has been a change in circumstances since the original offer and it is in the interests of justice.

A

Simon should immediately serve a notice of withdrawal. It would then be for Clinton to decide whether or not to serve notice of acceptance. If Clinton did serve notice of acceptance Simon should apply to the court for permission to withdraw the offer within 7 days of Clinton’s notice of acceptance. Simon would argue that the court should grant permission as there has been a change in circumstances since the original offer and it is in the interests of justice.

Correct. Simon’s Part 36 offer of £750,000 is now very likely to be too high as the injury is worth less than the Part 36 offer (in the light of the evidence in the recording and Clinton playing football for his local football team during which he scores a goal with an overhead kick). As soon as Simon serves the recording evidence (or realises that Simon has it) Clinton may very well accept the offer. In order to prevent acceptance Simon needs IMMEDIATELY to serve a notice to withdraw the offer. Clinton then has a choice whether to accept the offer or allow the withdrawal to take effect. If Clinton does not accept the offer at that stage the offer will be withdrawn at the expiry of the relevant period. If Clinton does accept the Part 36 offer then Simon will need to apply to the court to withdraw the offer within 7 days of Clinton’s notice of acceptance (CPR 36.10).

173
Q

Landsdown Limited (‘Landsdown’) is the claimant in a civil dispute. Gothic Limited (‘Gothic’) is the defendant. A trial is due to take place in 3 months’ time. The claim is for £500,000. Gothic has made an offer to Landsdown to settle the claim for £350,000. The offer was made pursuant to Part 36 of the CPR. Landsdown has rejected the offer.

The claim proceeds to trial. At trial, Landsdown is awarded £380,000. What is the impact of Gothic’s offer on costs?

Landsdown will get its damages and a split costs order. Gothic will have to pay Landsdown’s costs until the relevant period expired. Thereafter, Landsdown will have to pay Gothic’s costs plus interest.

No effect. Landsdown will get its damages and it will also most likely get its costs paid by Gothic pursuant to the normal rules on costs (CPR 44.2).

Landsdown will get its damages and a split costs order. Landsdown will have to pay Gothic’s costs until the relevant period expired. Thereafter, Gothic will have to pay Landsdown’s costs plus interest.

Gothic will most likely get its costs paid by Landsdown pursuant to the normal rules on costs (CPR 44.2).

A

No effect. Landsdown will get its damages and it will also most likely get its costs paid by Gothic pursuant to the normal rules on costs (CPR 44.2).

Correct. The claimant (Landsdown) has won. Importantly, Landsdown has obtained more than the offer of £350,000 (it has been awarded £380,000). In other words it obtained a judgment ‘more advantageous’ than the defendant’s (Gothic’s) Part 36 offer. A split costs order would only be appropriate if Landsdown had won, but failed to obtain a more advantageous judgment than Gothic’s Part 36 offer eg if Landsdown had been awarded £340,000.

174
Q

Where judgment against the defendant is at least as advantageous to the claimant as the proposals contained in a claimant’s Part 36 offer, which of the following IS NOT one of the stipulated consequences (the consequences apply unless the court considers it unjust).

C is awarded interest on costs from the end of the relevant period at a rate not exceeding 10% above base rate.

All the other answers are consequences.

C is awarded interest on the award at a rate not exceeding 10% above base rate for some or all of the period from the expiry of the relevant period.

C is awarded costs from the end of the relevant period to be assessed on the indemnity basis (in contrast to the usual basis - the standard basis).

C is awarded an additional amount based on a percentage of the award.

A

All the other answers are consequences

Correct. All the other answers are consequences (CPR 36.17).

175
Q

C has brought a breach of contract claim against D seeking damages of £140,000. 4 months before trial, D makes a Part 36 offer of £110,000. C does not accept this. At trial, C is awarded damages of £100,000. What is the likely impact of the Part 36 offer, if any?

No effect. C will get its damages and it will also most likely get its costs paid by D pursuant to the normal rules on costs (CPR 44.2).

C will get its damages and a split costs order. D will have to pay C’s costs until the expiry of the relevant period. Thereafter, C will have to pay D’s costs plus interest.

D will most likely get its costs paid by C pursuant to the normal rules on costs (CPR 44.2).

C will get its damages and a split costs order. C will have to pay D’s costs until the relevant period expires. Thereafter, D will have to pay C’s costs plus interest.

A

C will get its damages and a split costs order. D will have to pay C’s costs until the expiry of the relevant period. Thereafter, C will have to pay D’s costs plus interest

Correct. A split costs order is appropriate because C has won, but failed to obtain a more advantageous judgment than D’s Part 36 offer. The costs are therefore ‘split’ as set out in this answer, and D is also awarded interest on those costs. Note, this is the result, unless the court considers it unjust.

176
Q

Which of the following necessarily results in a binding decision?

Expert determination

Expert appraisal

Negotiation

Mediation

A

Expert determination

Well done. In expert determination, an independent expert reaches a binding decision on the matter referred to them. Expert appraisal does not result in a binding decision – the expert provides a non-binding assessment which might influence future settlement discussions. Neither negotiation nor mediation will result in binding outcomes unless they are successful, and even then the outcomes should not be considered ‘decisions’, because they are outcomes mutually agreed by the parties, not decided by a third party

177
Q

Which of the following is not / very unlikely to be an advantage of ADR?

It saves money for the parties

It allows a greater range of outcomes than court proceedings.

It is more private

It sets a binding precedent.

A

It sets a binding precedent.

Well done. A resolution reached by ADR will rarely, if ever, amount to a binding precedent. The other answers are often advantages of ADR, although whether they are advantages in any particular case depends on the facts of that case.

178
Q

When can parties agree to arbitrate?

After a dispute has arisen.

When entering into a contract, before any dispute has arisen.

After a dispute has arisen or when entering into a contract before any dispute has arisen.

A

After a dispute has arisen or when entering into a contract before any dispute has arisen.

Well done. Parties might include an arbitration clause when negotiating a contract, which provides for any dispute in relation to that contract to be referred to arbitration. However, even if the parties do not include such a clause, they can refer a dispute to arbitration once that dispute arises, if they both agree to this.

179
Q

How likely is it that a case is suitable for some sort of ADR?

Possible

Unlikely

Rare

Certain

Almost certain

A

Almost certain

Well done. The vast majority of cases are suitable for ADR, if ADR is appropriately chosen and timed.

180
Q

Which of the following best summarises the court’s powers in relation to ADR?

The court can encourage the parties to consider ADR and compel a party to engage in ADR.

The court can provide information about ADR, encourage the parties to consider ADR and compel a party to engage in ADR.

The court can provide information about ADR.

The court can provide information about ADR and encourage the parties to consider ADR.

A

The court can provide information about ADR and encourage the parties to consider ADR.

Well done. The court cannot compel parties to engage in ADR, although it can impose sanctions on a party that unreasonably refuses to engage in ADR.

181
Q

Which of the following IS NOT part of the guidance given by the court as to how the court will approach the question of costs where a successful party has refused to engage in ADR?

The starting point is that a party who has refused to engage in ADR should pay the costs from the refusal onwards, unless factors point to a different order.

A relevant consideration is the extent to which settlement methods other than the one ultimately refused have been attempted.

The burden of proof will be on the unsuccessful party to show the court why it should depart from the general rule on costs to deprive the successful party of some or all of its costs on the grounds that it refused to agree to ADR.

A refusal is more likely to be reasonable if ADR had little prospects of success.

A

The starting point is that a party who has refused to engage in ADR should pay the costs from the refusal onwards, unless factors point to a different order.

Well done. This statement is incorrect. The contrary is the case – it is for the unsuccessful party to show the court why it should depart from the general rule that the unsuccessful party should pay the successful party’s costs. The extent to which other settlement methods have been attempted, and the prospects of ADR succeeding, are also both relevant considerations.

182
Q

Which of the following IS BEYOND the court’s powers in relation to ADR?

To order a party to attend (but not necessarily to make an offer at) a mediation.

To order a party to serve a witness statement if it refuses an opponent’s ADR proposal.

To penalise a party in costs for refusing to engage in ADR.

To order a stay of proceedings in order that the parties can explore ADR.

A

To order a party to attend (but not necessarily to make an offer at) a mediation

Well done. This is beyond the court’s powers.

183
Q

True or false: a party’s failure to propose ADR at any stage of a claim is likely to be treated by the court in the same way as a refusal to engage in ADR proposed by the other party.

False

True

A

False

Well done. Failing to propose ADR is generally less likely to be considered conduct worthy of criticism than refusing to engage in ADR, although either can attract criticism and sanctions.

184
Q

Who can discontinue proceedings?

The claimant

The defendant

Either the claimant or the defendant.

A

The claimant

Correct. Discontinuance is available to a claimant, and only a claimant.

185
Q

A claimant discontinues proceedings against Defendant A but not against Defendant B. What are the costs consequences (unless the court orders otherwise)?

The court will determine liability for costs upon the discontinuance.

The claimant will be required to pay the costs of Defendant A and Defendant B up to discontinuance.

The court will determine liability for costs upon the determination of the claim against Defendant B.

The claimant will be required to pay the costs of Defendant A up to discontinuance, but not those of Defendant B.

A

The claimant will be required to pay the costs of Defendant A up to discontinuance, but not those of Defendant B.

Correct. The claimant is liable for costs relating only to the part of the proceedings which they are discontinuing (CPR 38.6).

186
Q

When does discontinuance take effect?

Upon the court order discontinuing the claim.

Upon payment by the claimant of the defendant’s costs.

Upon service of the notice of discontinuance

Upon filing of the notice of discontinuance.

A

Upon service of the notice of discontinuance

Correct. (CPR 38.5(1)).

187
Q

Discontinuance is usually achieved by….

Filing a consent order.

Making an application to court.

Filing a notice at court and serving it on all other parties.

Filing a notice at court.

A

Filing a notice at court and serving it on all other parties.

188
Q

Your client, a supermarket, is about to enter into a contract with a new supplier of almonds and other nuts based in the USA. The supplier has proposed including an arbitration clause in the contract to resolve any disputes that may arise in future. The arbitration will have its legal seat in England. Your client is more familiar with litigation in the English courts and has generally been happy for its previous contractual disputes to be adjudicated with legal expertise to provide correct legal solutions. It has asked for your advice on arbitration. Which one of the following potential benefits of arbitration is likely to be most attractive to your client in its dealings with this supplier?

Arbitration is likely to be cheaper for your client than litigation in the English courts.

Arbitration is likely to enable a legally correct solution to be achieved.

Arbitration allows for easier international enforcement than litigation in the English courts

Arbitration allows your client to have some input into the process

The arbitrator can be experienced in the area of food supply.

A

Arbitration allows for easier international enforcement than litigation in the English courts

Correct. It is often easier to enforce an arbitration agreement (the reason for this is that the vast majority of countries are signatories to the New York Convention which means arbitration awards can be enforced internationally, whereas court judgments are more difficult to enforce and usually depend on there being a specific agreement between the two countries). This is important to the client given the supplier is in another jurisdiction because any judgment/award needs to be enforceable. In relation to the other answers, arbitration is rarely cheaper than litigation if the parties follow a very long and complicated process and the client has not said it is concerned about costs. Whilst the arbitration process allows for some flexibility in the process (subject to the mandatory rules in the Arbitration Act 1996 which provides a framework where the seat of the arbitration is in England/Wales), this is not a benefit of arbitration that is likely to be ‘most attractive’ to the client on the facts. Whilst arbitrators can have more expertise if available in the particular field (and this is a major benefit of arbitration), in this case, it is not something the client has mentioned it is concerned about. In fact, the client has previously been happy with judicial adjudication in the courts. Finally, arbitration allows for a legally correct solution to be achieved if the parties agree this, but so do court proceedings. So again this is not a benefit of arbitration that is likely to be ‘most attractive’ to your client.

189
Q

Your client is a producer of high-quality furniture. One of its long-standing suppliers provided it with sub-standard leather to cover its sofas. This resulted in the recall of the sofas and significant losses for your client. Your client has recently commenced court proceedings against the supplier to recover these losses. The supplier has proposed that the parties try to resolve the dispute by mediation and your client asks for your advice. Your client is not yet concerned about the increasing costs of the litigation as it has put aside a small fund to pay for it. The client believes it has a strong case so that any legally correct solution would be decided in its favour. It would like to continue to work with the supplier in future. Which of the following is most accurately describes the greatest benefit of mediation to your client?

Mediation is a less adversarial process than court proceedings so it will enable the parties to maintain their commercial relationship in future.

Mediation will shorten the time spent on the dispute because the mediator can make a decision which will bring the dispute to a conclusion.

Mediation is more likely to provide a commercial resolution.

Mediation is confidential.

Mediation will save your client costs.

A

Mediation is a less adversarial process than court proceedings so it will enable the parties to maintain their commercial relationship in future

Correct. Mediation is less adversarial than court proceedings and so is a good choice if the parties wish to maintain their commercial relationship after the dispute has been resolved. In terms of the other answers, mediation will save costs if successful (although probably not otherwise), but this does not appear to be a key concern for your client. It will shorten the dispute if successful, but not because the mediator can make a decision – they cannot, they simply facilitate settlement. One of the benefits of mediation is that it is more likely to be a commercial resolution rather than a legally correct one, but this is not a clear benefit to our client who believes it has a strong case that will be decided in its favour. Mediation is indeed confidential, but your client has not identified this as a significant concern – perhaps the product recall has already publicised this issue so that this is no longer a critical concern

190
Q

The Claimant issues proceedings against Company A and Company B for breach of contract. Company A and Company B instruct their own solicitors and serve separate defences. Company A and Company B had worked together in a joint venture partnership and initially appeared equally liable for the breach of contract. On receipt of Company B’s defence, it becomes clear to the Claimant that there was no joint venture agreement and on re-assessing the documentary evidence, the Claimant only has a contract with Company A. What is the best course of action for the Claimant to take?

Apply for summary judgment against Company A

Apply to strike out the particulars of claim against Company B

Discontinue the claim against Company B

Apply for a stay of proceedings

Discontinue the claim against Company A and company B and issue new proceedings against Company A

A

Discontinue the claim against Company B

Correct. The Claimant can discontinue its claim against Company B by filing a notice of discontinuance at court and serving a copy of it on every other party to the proceedings (CPR 38.3(1)). Where there is more than one defendant, the notice of discontinuance must specify against which Defendant(s) the claim is discontinued (CPR 38.3(4)). The Claimant is likely to be required to pay Company B’s costs, but better to face that now than after losing against Company B at trial. In terms of the other answers, discontinuing against Company A and Company B and issuing new proceedings against Company A is going to be much more costly, and could be an abuse of process. A stay of proceedings would not solve the problem, nor are there grounds for summary judgment. The Claimant would not apply to strike out its own statement of case.

191
Q

You represent a claimant company which is bringing an action alleging negligence against its former surveyors, the defendant. The defendant makes an offer pursuant to Part 36 in an attempt to settle the action before the issue of proceedings. The next day, the claimant issues proceedings, but shortly afterwards decides it should accept the defendant’s offer.

Which one of the following is the best advice to the claimant should it wish to accept the defendant’s offer?

Provided that the claimant accepts the offer within the relevant period, it will be entitled to damages and all of its costs up to the expiry of the relevant period.

The claimant must accept the offer within the relevant period or else the offer will automatically be withdrawn at the end of that relevant period.

The claimant may accept the offer within the relevant period and obtain damages and costs to the date the notice of acceptance is served.

The claimant may accept the offer within the relevant period, but as the offer was made before proceedings were issued, Part 36 will not apply.

The claimant can accept the offer by filing a written notice of acceptance at court.

A

The claimant may accept the offer within the relevant period and obtain damages and costs to the date the notice of acceptance is served.

Correct. A Part 36 offer may be made at any time, including before the commencement of proceedings (CPR 36.7). Pre-issue Part 36 offers have the usual Part 36 consequences, including recoverable pre-action costs, upon acceptance after issue of proceedings (or upon judgment being given).

192
Q

A claimant is bringing a civil action against a defendant for £400,000. After exchange of witness evidence, the defendant serves on the claimant a Part 36 offer to settle in the sum of £300,000. The claimant rejects this offer and the case proceeds to trial where the claimant is awarded £300,000 (all figures are inclusive of interest). Which ONE of the following statements best describes the likely costs consequences?

There are no part 36 costs consequences, because the claimant rejected the defendant’s offer within the relevant period.

The claimant will pay the defendant’s costs before the expiry of the relevant period, and the defendant will pay the claimant’s costs from the expiry of the relevant period, pursuant to CPR Part 36, because the claimant beat the defendant’s offer.

The claimant will get its costs pursuant to the general rule because it has been successful at trial.

The defendant will pay the claimant’s costs before the expiry of the relevant period, and the claimant will pay the defendant’s costs from the expiry of the relevant period, pursuant to CPR Part 36, because the claimant failed to beat the defendant’s offer.

The claimant will have to pay the defendant’s costs of the proceedings pursuant to the general rule because the claimant has refused a reasonable offer.

A

The defendant will pay the claimant’s costs before the expiry of the relevant period, and the claimant will pay the defendant’s costs from the expiry of the relevant period, pursuant to CPR Part 36, because the claimant failed to beat the defendant’s offer

Correct - This is a Defendant’s part 36 offer. The claimant has failed to obtain judgment more advantageous than the offer (CPR 36.17(1)(a)) as it has obtained a judgment the same as the offer. This triggers Part 36 and it is likely that a split costs order (CPR 36.17(3)) may apply where the claimant will get its costs from the defendant up to the date on which the relevant period expires and the defendant will get its costs from the claimant thereafter. Note, however, that this outcome is not guaranteed – the court will not impose these Part 36 consequences if it considers them to be unjust.